You are on page 1of 88

2

Collection of NEW SLE (Saudi License Exam) Questions


(Updated from start of April2010 till 15 of June2010)
Source : http://www.cksu.com/vb/t100804/
....#0234+
* ,-#"./0‫"!؛وא‬#$% &'(.‫  
د‬،‫ א אא‬....
KKK
KKK‫
 د‬KKK‫
 د‬KKK‫
 د‬KKK‫
 د‬KKK‫د‬KKK
 ، !‫א‬#$%&،' ()*‫א‬+ ,‫دم‬,/&ً‫א‬/1W

K8
7 9:;‫< د‬%&‫و‬،>)?‫و‬1@A‫ل‬56 ‫و‬1BC‫א‬+ ,23‫ل‬56

1) The body secretes antibody against antigen using which cells


-T lymphocyte
- B lymphocyte++++

2) In diabetic patient , periodontium affected by which cells


- Neutrophils
- Macrophages++++

3) When taking an x-ray to pregnant lady we use all of this method except:
- digital x-ray
- high sensitive film
- 16 inch / long cone / paralleling tech
- 8 inch / short cone / bisecting angle tech++++
- lead apron with thyroid collar

4) Best measurement of periodontitis by:


- pocket depth
- bleeding
- attachment level++++

5) Proximal caries should be opened when


- confined within enamel
- pass DE junction
- dentine lateraly
- all of the above++++

6) In a study, it should
- Protect you against role of the statistician
- Protect you against legal risks++++

Page | 1
- Protect you against physical risks

7) Ethics of the study include all of the following except:


- privacy of all subjects
- informed consent may be required or not
- object if the subject refuse to take part of the study++++

8) In clinical research
- no difference between blind & double blind.
- if there's need of intervention.++++

9) Pit and fissure sealant:


-new erupted teeth
-deep fissure and pits in molars
-proximal caries
-a & b++++

10) Ugly duckling stage:


-9 to 11 years old++++
-13 to 15 years old
-7 to 9 years old

11) Component of Gutta percha:


-50% Gp & 50% ZOE
-20% Gp & 70% ZOE++++ (Exact is 73%ZOE, 23% Gp, 4-6% others.)

12) All are irrigation for canals except:


-NaOCl
-RC prep++++ (RC Prep = EDTA)

13) For post preparation we should leave ……mm of GP:


-10mm
-5mm++++ (apical 1/3 of the obturated canal)

14) Enamel caries :


- outer zone free of….
- outer zone contain high pores

15) Proximal caries confined to enamel:


- prevention++++
-observation
-restore with G I++++

Page | 2
16) In community diagnosis and treatment program:
-water fluoridation
- diagnose, prevent, treat++++

17) Pass through parotid gland:


-facial nerve++++
-facial arteries
-external carotid veins

18) Porcelain shrinkage after firing:


- 1-5%
-5-10%
-10-20%++++ (exactly 13.3%)

19) the cement under MOD amalgam have this character:


a) high modulus of elasticity++++
b) low modulus of elasticity
c) the high modulus of elasticity prevent of bonding and decrease tensile strength.
d) both a &c

20) Radiographic radioulucency in the interradicular area:


a) invasion of furcation. (will be present with other clinical manifestations)
b) periodontal abcess (not enough radiolucent)
c) periodontal cyst++++

21) Examination of pt health by the dentist:


- to know the patients health
- to know what medications to give
- to know general health data
- all of the above++++

22) Statements true or false:


• RCT abutment of FPD have higher risk for fracture.
• abutment which have RCT in cantilever FPD have higher susceptibility to fracture
-1st is true,2nd is false
-1st is false,2nd is true
-both are false
-both are true++++

Page | 3
23) Both glass ionomer & polycarpoxylate cement contain:
-polyacrylic acid++++
-ZO powder

24) Most frequent cause of fainting in dental office:


-vaso-vagal shock++++
-diabetes
-fear++++

25) Loss of consciousness most frequent cause:


-syncope++++
-CO2

26) Most common cyst in the oral cavity:


-radicular cyst++++
-peridontal cyst

27) Ostomylitis most in:


-maxilla
-mandible++++

28) Factors delay healing of wound:


-infection
-torn wound edges
-strain
-poor suturing
- all of the above++++

29) Dry socket happens after:


-24 hours
-3 to 5 days++++
-1 week
-2 weeks

30) Compared to herpetic ulcers... aphthous ulcers are:


-smaller in size
-in mucosal lining++++
-leave scars

Page | 4
31) In avulsion, most important factor that affect reimplantation :
-contaminated roots
-time since the avulsion++++

32) nicotine stomatitis:


an oral pathological condition that appears in the hard palate of the mouth as a white lesion. It is not
considered to be premalignant and results from tobacco smoking (especially pipes or cigars. found in
men over 45 years of age, it is characterized as a "fissured" or "dried mud" appearance from excess
keratin production by cells. The palate may appear gray or white and contain many papules that are
slightly elevated with red in their center. Furthermore, the teeth may be stained brown or black.

33) Dentinogenisis imperfecta have all signs except:


- broken enamel
- blue sclera
- broken bone
- supernumerary teeth++++

34) Most sign of fracture of mandible:


- nose bleeding
- malocclusion++++
- parasthesia

35) Hairy tongue is elongation of:


- fungiform papilla
- filiform papilla++++

36) What supply the gingival buccal tissue of premolars,canines and incisors:
-long buccal
-inferior alveolar nerve
-superior alveolar nerve
-mental nerve <branch of IAN>++++

37) Drainage of tip of the tongue:


-submandibular lymph nodes
-submental++++

38) Cementum in cervical 2/3 have:


-acellular intrinsic fiber
-acellular extrinsic fiber++++
-cellular mixed fibers
Page | 5
-intermediate cementum

39) Periodontal attachment contains:


-epithilum, sulcus, connective tissue++++

40) Pins are inserted into:


-enamel
-dentin++++
-DEJ
-all

41) After etch enamel and bond it with 5th generation the strength of…… :
- 5-10Mp
- 25Mp++++
- 30Mp
- 100Mp

42) LA in mg if epinephrine 1:100 000 in 2% xylocaine:


-0.01 mg
-1.8 mg
-3.6mg++++ (Dental decks p.1870)

43) Composite restoration that was matching in shade ,after one week it became much lighter..the
reason could be:
-light started photoinitation
-absorption water
-shade selected after rubber dam++++

44) disadvantage of digital x-ray are all, except:


-large disk space Storage
- clarity and resolution++++ (ONE OF THE MAIN ADV’s of digital x-ray)
-expensive

45) treatment of fungal infections:


-Amphotrecine B (topical treatment)
-tetracycline
-Flucnazole (Systemic treatment)
-nystatin (topical treatment)

Page | 6
46) properties of ideal endo obturation material are all except:
-radiolucent++++

47) vibrating line :


-b/w hard & soft palate
-b/w mobile and non mobile soft tissue of palate++++

48) pt has hyperventilation in clinic most cause:


- reduced amt of CO2
-increase amt CO2
-anxiety++++

49) all relate to retention of maxillary complete denture except:


-tongue movement++++
-type of saliva

50) most difficult of extract:


-mand. 3rd molar with mesioangular fused roots
-mand 3rd molar with distoangular angulation with divergent curve roots++++

51) Very important part in endo treatment:


-complete depridment of the canal++++

52) Perio endo lesion the primary treatment:


- endo treatment++++ (desck page 216)
-deep scaling and root planning

53) Provisional luting cement:


-prevent restoration from dislodgement++
-sealing++++

54) contra indication to extraction:


-cardiac pt
-previous recent radio therapy++++

55) base of the flap should be wide for:


-healing
-better blood supply to the wound++++

Page | 7
56) Supra calculus all true except:
- easy to detach
- has component of saliva++++

57) to prevent perio problem MOST effective method is:


-community program
- Frequent removal of plaque++++
-patient education

58) thickness of luting cement


-100 micrometer
-40micrometer++++
-1mm

59) formacresol used in:


- full concentration
- 5th concentration
- 1/5 concentration++++

60) zinc phosphate cement:


-mechanical attachment++++
-Chemical

61) traditional Glass ionomer:


- mechanical bonding
-Mechanical chemical bonding ++++
-acid/base reaction

62) pontic design of an FPD:


-Same size buccolingually of the missing tooth
-smaller than missing buccolingually++++
-wider buccolingually
-none of the above

63) Maryland bridge:


-use with young ++++
-to replace single missing tooth

64) false negative response of an electric pulp test given:


-after trauma ++++
Page | 8
-periodontal disease
- in teenager

65) young with open apex examination test:


-reliable
-non reliable++++
-none of the above

66) primary teeth had trauma, tooth change in color become white yellowish ,what should u tell the
parents:
-pulp is dead Hemmoragic pulp : Reddish in color
-inflammation of pulp Calcification / partial or complete obstructed canal : Yellowish
-calcification of dentin++++
Necrosis : dark gray color
-b&c

67) best media for the avulsed tooth:


-milk++++ (the best media is the buccal sulcus of the same patient)
-water

68) if enamel caries passing half of enamel:


-leave it
-restoration ++++

69) biological depth:


-crestal bone to gingival sulcus++++

70) Pt wearing denture 5 year ago came with white patches on lower ridge what is the best to be done:
-surgical removal of patches.
-tell pt stop wearing lower denture & recheck after2 weeks++++.
-no treatment.

• The glenoid fossa = the mandibular fossa.


71) Glenoid fossa is found in:
• The mandibular fossa: a depression in the temporal
1/ orbital cavity bone that articulates with the condyle of the
2/nasal cavity mandible and is divided into two parts by a slit.
3/ middle cranial fossa
d) temporal bone++++

72) The spread of odontogenic infection is based on:


1/ host defense
2/ virulent of microorganism
Page | 9
3/ No. of bacteria
4/ all++++

73) Knife ridge should be tx with :


1/relining soft matiral++++
2/ maximum coverage of flange
3/ wide occ. table
4/all

74) AH26 composed of:


1/ZOE
2/ CaOH
3/ epoxy resin++++
4/ hydrogen peroxide

75) The splint period require for avulsed teeth:


- 6w
- 8w
- 1-2 month
- None of the above (the maximum time to splint is 2 weeks) ++++

76) child 3 years old came to clinic after falling on his chin, you found that the primary incisor
entered the follicle for the permanant inicir what you will do
a)surgical removal of the follicle
b)leave it
c)surgicall removal of the primary incisor++++

77) tongue develope from:


1/mandibular arch &toburclum impar++++
2/1st bronchial arch

78) isolated pocket in :


1/ vertical root fracture++++
2/palato gingival groove
3/endo origin lesion
4/ all

79) perforation during endo space preparation wht is the most surface of distal root of lower molar
will have tendency of perforation:
1/ M SURFACE++++
Page | 10
2/ '' D
3/''B

80) crown and root perforation:


1/ respond to MTA
2/ use matrix with hydroxiapatite and seal with G I
3/1&2++++
4/root canal filling

81) 3rd generation of apexo locator:


- use with all pt
- need more research
-increase chair time
-decrease radiographic film need++++

82) acceptable theory for dental pain :


1/hydrodynamic++++
2/fluid movement
3/ direct transduction

83) removing of dentine in dangerouse zone to cementum is :


1/ perforation
2/ledge
3/stripping++++
4/zipping

84) tracing of GP used for:


1/to detect source of oeriapical pathosis++++ (to follow the GP through the sinus tract in the x-ray)
2/acute periapical periodontitis
3/ periodental abscess
4/ non

85) occlusal splint device:


1/ used during increasing vertical dimension++++
2/allative muscle of mastication
3/ occlusal plane CR/CO
4/ALL

86) silane coupling agent:


1/ used with porcelain to enhance wetability of bonding++++
Page | 11
2/ used with tooth and porcaline

87) when increase vertical dimension you have to:


1/ increase minimal need
2/construct anterior teeth first thenposterior teeth
3/ use provisional crown for 2 months++++
4/all

88) amalgam keep under:


a fixer++++
b-developer
c- water

89) the irrigation material used to kill the E.Faecalis bacteria:


a-Na oH
b-MTA++++
c-saline

90) hand instrument which we used to make internal angle retentive groove and preparation of cavity
wall in the cavity is:
a-angle former++++ (source : Art & Science)
b-chisel
c-file
d-enamal hatched

91) What is the no of the pharyngeal arches?


1/ 6++++

92) To detect the occlusal plan which of the following used?


1/ Frankfort plan
2/ ala-tragus line++++

93) Streptococcus mutans cause caries & this disease is?


1/ eipdemic
2/endemic++++
3/isolated

94) pt came to the clinic complaining from pain related to swelling on maxillary central incisor area
with vital to under percussion?
1/ periapical cyst
2/incisive cyst( nasopalatin cyst)++++
3/ globulomaxillary cyst

Page | 12
4/ anuyrsmal bone cyst

95) pt came to the clinic & u revealed under medical history he had chronic renal failure; he used to
do hemodialysis the treatment should be:
1/ before one day of dialysis
2/ on the day of dialysis
3/ after one day of dialysis ++++
4/ after one week of dialysis

96) pt taken heparins he should do surgery after :


1/ 1 Hr
2/ 2 Hr
3/ 4 Hr
4/ 6 Hr++++ Treacher Collins Syndrome is characterized by :

97) Treacher – Collins syndrome is mainly: 1. incomplete growth of mandible (retrognathic).


1/ mandibular retrognathia ++++ 2. slinting of eyes corner.
2/ loss of hearing 3. Absent zygomatic arch / malar bone
4. malformed or absent ear auricles with weakened or
98) the most common injures in child is: (in 50%) loss of hearing.
1/ tooth ++++
2/ root
3/ intrusion of the tooth inside the socket well

99) The power tooth brush invented in:


1/ 1939 ++++
2/ 1949
3/ 1929
4/ 1959

100) location to give inferior alveolar nerve block the landmarks are:
1/ ptergomandibular raphy
2/ cronoid notch
3/ all of the above ++++

101) what kind of periodontal probe is used in the furcation area:


1/ WHO
2/ Nabers probe ++++
3/ UNC 15
4/ Michigan

102) what kind of suture used under the immediate denture:


1/ horizontal matter suture
2/ vertical matter suture
3/ interrupted suture

Page | 13
4/ continous locked suture ++++

103) Ideal properties of RC filling material is the following except:


a) Radiolucent in radiograph++++
b)Not irritate the surrounding tissue
c) Easily removable when retreatment is necessary
d)Stable and less dimensional change after insertion

104) During instrumentation sudden disappear of root canal due to :


a) Bifurcation of main canal ++++
b) Apical perforation
c) Calcification

105) After examination of the tooth


a)start local anesthesia
b)excavate the dentin
c)testing the tooth with the pulp tester++++

106) Incipient caries in the old patients is MOSTLY due to:


a)smoking
b)saliva
c)xerostomia++++

107) The best restoration for the maxillary central incisor that has received root canal treatment
through a conservative accsess opening would be:
a) post retained metal crow
b) post retained porcelain jacket crown
c) composite resin++++

108) In post and core , it is important to:


a)end the core at contra bevel to produce ferrule effect
b)end the core at the cervical finish line
c)the core take the same shape of a natural toorh
d)the core take the same shape of prepared tooth
- a only
- b&d
- a&c
-a&d++++

109) Thickness of the porcelain in metal-ceramic restoration


a) 1.7 – 2.3mm
b) 0.5 – 1.5 cm
c) 0.2 mm
d) 0.05 – 0.15 cm (0.5 mm- 1.5mm)++++

Page | 14
110) Patient came to your clinic with dull pain in the #6 ,no response to the pulp tester, in
radiographs it shows 3mm of radiolucency at the apex of the root
Diagnosis is
a)chronic apical periodontitis++++
b) acute apical periodontitis
c)acute periodontitis with abscess

111) Composite for posterior teeth


a) Macrofilled + fine filler
b) Hybrid + rough filler ++++

112) Pulp stone can be the following except:


a) present freely in the pulp
b)cause discomfort & pain to the patient
c)In radiographs,Small spheroidal radioopaque
d)False stone occurs due to destrohpic dentin ++++

113) The optic foramen canal is a part of:


a)Frontal bone
b)Sphenoid bone++++
c)Esthmoid bone

114) In growth, cartilages may give rise to bones:


a)true++++
b)false

115) Adjustment of proximal contact: with


a) Shim stock
Note:
b) Pencil
Shim Stock : Wedge / Spacer.
c) Dental floss++++
d) c &d

116) Best core material receiving a crown on molar


a)amalgam++++
b)reinforced glass ionomer
d)composite

117) Oral lesion of lichen planus usually appears as:


a)white streaks++++
b)Red plaques
c)small shallow ulcers
d)papillary projections
e)bullae

Page | 15
118) Radiographic diagnosis of bone destructive lesion in the mandible without evidence of
bone formation is:
a)Osteomyelitis
b)Malignancy++++
c)Fibro-osseous lesion
d)Fracture
e)Osteoradionecrosis

119) Calculus induce further periodontal lesion due to:


a) Directly stimulates inflammation
b)more plaque adhere to it++++

120) Most coomon site to drain pus is:


a) mandibular central incisors
b)mandibular canines
c) mandibular first molar ++++

121) First of pharyngeal arch is:


a)Maxilla
b)Mandible++++
c)hyoid

122) The nmber of pharyngeal arches is:


a)One
b)Two
c)three
d)Six++++
e)Seven

123) The naso palatine bone forms a triangle will be parallel to an imaginary lines extended
between cemento-enamel junctions of adjacent teeth
a)true++++
b)false

124) *Bross brushing has the advantage of the bristles enters in the cervical area .
*It is recommended for all patients.
a)both statements are true
b)both statements are false
c)first is true ,second is wrong++++
d)first is wrong , second is true

125) Chronic suppurative periodontitis


a) Patient complains of moderate pain
b) Fistula with pus discharge++++
Page | 16
c) Pulp polyp in open caries

126) Complete denture poorly fit and inadequate interocclusal relation


a) relining
b) rebasing
c) new denture ++++

127) The incisal guidance on the articulator


a) same as condyler guidance
b) compensating for the overbite and overjet++++
c) compensating for curve of spee

128) The primary drainage(or spread of infection) of the mandible is the submental lymph node:
a)true
b)false++++

129) Most common tooth which needs surgical extraction


a)mandibualr third molar++++

130) The nerve which supply the tongue and which may be anesthetized during nerve block
injection
(Lingual Nerve branch of
a) V ++++
b) VII the Inferior Alveolar
c) IX Nerve of the Trigemenal
d) XII N (Cranial V)

131) when removing moist carious dentin which exposes the pulp , dentist should
1- do direct pulp cap (is done upon accidental exposure)
2- do indirect pulp cap (Very deep and close from pulp with Sclerotic/affected hard dentine)
3- prepare for endo ++++ (As long as there’s infection remaining)

132) In prevention of dental caries, the promotion of a healthy diet is:


1- low effective measure
2- moderately effective measure++++
3- high effective measure

133) Treatment of cervical caries in old patients with a temporary restoration is best done by:
a)Glass ionomer++++
b)composite resitn

134) Squamous cell carcinoma is


a) multifactorial++++

135) Most used sugar substitute:


a) Sorbitol
b) Mannitol
Page | 17
c) Xylitol++++

136) Follow up of RCT after 3 years , RC failed best treatment is to:


a) Extraction of the tooth
b) Redo the RCT++++
c) Apicectomy

137) Small access opening in upper central incisor in necrotic pulp leads to
a) Complete removal of the pulp tissue
b) Incomplete removal of the pulp ++++
c) better Conservation

138) Class II amalgam restoration with deep caries the patient comes with localized pain
related to it after 3 months due to:
a) pulp horn exposure
b) over occlusion
c) moisture contamination during the restoration++++

139) Early Squamous cell carcinoma of the oral cavity presents as


a) Vesicle
b) A sessile mass
c) Red plaque
d) growing cauliflower like lesion ++++ <appearance of Sq. cell carcinoma>

140) In recording mand-max jaw relation, the best material used without producing pressure is:
a) Wax
b) Compound
c) zinc oxide & eugenol paste ++++

141) acute abscess is:


a) Cavity lined by epithelium
b) Cavity containing blood cells
c) Cavity containing pus cells ++++
d) Cavity containing fluid

142) The most close tooth to the maxillary sinus


a)maxillary 1st , 2nd molar++++
b)maxillary bicuspids
d) none of the above

143) 33 years old female patient. came with slow growing swelling in the angle of the mandible
radiograph show radio-opaque with radio- lucent border diagnosis
a) Osteoma
b) Osteosarcoma++++
c) Cementoblastoma

Page | 18
144) Reliability of the measurements reflects that property of the measurements which
a) Measures what is intended to be measured
b) Produces repeatedly the same result under a variety of conditions
c) Detects reasonably small shifts, in either directions, in group condition
d) All of the above++++

145) Balanced occlusion refers to


a) The type of occlusion which allows simultaneous contact of the teeth in centric occlusion only
b) The type of occlusion which allows simultaneous contact of teeth in centric and eccentric jaw
positions ++++
c) A type of occlusion which is similar to the occlusion of the natural teeth

146) In recent years there has been evidence that the prevalence and intensity of the caries
attack has been diminished in the more economically developed countries mainly because of the
widespread use of
a) Artificial water fluoridation++++
b) Fluoride tooth paste
c) Dental health education programs ++ (2nd most important)
d) Individualized oral hygiene instruction

147) In terms of caries prevention the most effective and the most cost effective method is
a) Community based programs++++
b) Privately based programs
c) Individually based programs

148) In countries with higher annual population growth rates, the need for community based
preventive programs would be greater for
a) Dental caries ++++
b) Periodontal disease
c) Dentofacial anomalies
d) Dental fluorosis

149) Occlusal rest function:


a) To resist lateral chewing movement
b) To resist vertical forces ++++
c) Stability
d) Retention

150) Which of the following may cause gingival enlargement


a) Phenytoin (Dilantin) ++++
b) Cyclosporine
c) Nifedipine
d) Aspirin

Page | 19
e) None of the above

151) What is the best instrument used for removing unsupported enamel at the gingival wall of
Class II
a) Chisel
b) Hatchet
c) Gingival marginal trimmer ++++

152) Dental caries


a) Is transmissible disease
b) Is worldwide in distribution but uneven in intensity
c) Can be prevented
d) All of the above ++++
e) None of the above

153) Radiographic diagnosis of bilateral expansile radioopaque areas in the canine premolar
area region of the mandible is
a) Hematoma
b) Remaining roots
c) Torus mandibularis ++++
d) Internal oblique ridge
e) Genial tubercle

154) Mandibular foramen in children in comparison with it in the adults


a) Above the occlusal plane
b) Below the occlusal plane++++ (5 mm exactly below the occ. plane)
c) anterior position

155) The choice of LA tech. depends on Note that the Tech. itself is dependant on the
a) Diameter of nerve bone type (compact/cancellous).
b) Structure of bone ++++ while the type of the LA itself (with or
c) Number of branches without Adrenaline) is acc. to the patient
d) Type of LA agent chemistry systemic condition/health.

156) Dental plaque is composed mainly of


a) Bacteria ++++
b) Inorganic material
c) Food

157) Management of tuberosity fracture during extraction of maxillary molar is


a) Replace and suture (ONLY IF PERIOSTEUM IS PRESENT)
b) Remove and suture with primary healing (IF NO PERIOSTEUM TO AVOID NECROSIS)
c) Replace and suture intra alveolar by wire++++
d) Remove and leave it to heal

Page | 20
158) the retainer of rubber dam
a) 4 points of contact 2 buccally and 2 lingually without rocking+++
b) 4 points of contact 2 buccally and 2 lingually above the height of contour
c) 4 points of contact 2 mesially and 2 distally
d) 2 points

159) In hairy tongue, which taste buds increase in Length:


a)Filiform++++
b)Fungi form
c)Foliate
d)Circumvallates

160) the distal palatal termination of the maxillary complete denture base is dictated by the:
a- tuberosity
b- fovae palatine
c- maxillary tori
d- vibrating line++++
e- posterior palatal seal

161) in registering the vertical dimension of occlusion for the edentulous pt, the physiological
rest position:
a) is equal to the vertical dimension of occlusion
b) may be exceeded if the appearance of the pt is enhanced
c) is of a little importance as it is subjected to variations
d) must always be greater than vertical dimension of occlusion++++

162) Which of the following is a benign epithelial neoplasm?


a)Rhabdomyoma
b)Keratoacanthoma++
d)Fibroma
c)Lipoma
d)Granular Cell tumor

163) To a great extent, the forces occurring through a removable partial denture can be widely
distributed and minimized by the following methods:
a)proper location of the occlusal rests++++
b)selection of lingual bar major connector
c)developing balanced occlusion
d) all of the above

164) The posterior seal in upper complete denture serves the following functions
a) It reduces the patient discomfort when contact occurs between the dorsum of the tongue and
the posterior end of the denture base
b) Retention of the maxillary denture++++
Page | 21
c) It compensates for dimensional changes which occur in the denture base
d) all of the above

165) In growth, cartilages may give rise to bones:


a)true++++
b)false

166) The most common fissural abnormality is


a) cleft palate
b) cleft lip (least common)
c) cleft lip& palate ++++ (most common & most complicated) (Source: Peterson’s page 841)

167) 7 years patient come with untreated trauma to tooth that became yellow in color what
should you tell the parents
a) pulp is dead
b) pulp become calcified
c) the tooth will resorb normally

-a and b
-a and c++++
-all of the above

168) Inferior orbital fissure located:


-Lateral wall of the orbit & floor of the orbit.++++

169) Provisional restoration for metal ceramic abutment is


a) aluminum sheet
b) stainless steel crown
c) ZnO
d) tooth colored polycarbonate crown++++

170) In hypertension patient the history is important to detect severity


a) true++++
b) false

171) In 6 week intra-uterine life the development start. the oral epithelium is stratified
squamous epithelium will thickened and give dental lamina
a) true++++
b) false

172) When take x ray in upper premolar to locate lingual root using mesial shift it will appear
a) distal
b) buccal
c) lingual
d) mesial++++

Page | 22
173) A question about miswak:
use toothbrush after every meal and meswak before praying and if you out side home use
meswak++++

174) Hypercementosis and ankylosis is seen in


a) padget’s disease++++
b) monocytic fibrous dysplasia
c) hyper-parathyroidism

175) the infection will spread cervicaly in infection from


a) lower incisors
b) lower premolars
c) lower 2nd and 3rd molars+++
d) upper incsisors

176) open bite is seen in


a) lefort 2
b) unilateral condylar fracyure
c) bilateral condylar fracture++++

177) the first cervical vertebra is axis


a) true
b) false++++ (it’s Atlas)

178) in intra uterine life the mandible start to develop but clavicle starts before it
a) true++++
b) false

179) in maxillary upper 6, 4th canal is in


a) mesiobuccal++++
b) distobuccal
c) mesio lingual
d) lingual

180) osteoradionecrosis is more in


a) maxilla
b) mandible++++
c) no difference

181) in inflamed mucosa due to wearing denture, when to do new denture


a) immediately
b) after week
c) put tissue conditioning material and wait until the tissue heal and take impression after++++

Page | 23
182) schick intradermal test used in
a)Tuberclosis
b)tyhpoid
c)diphtheria++++

183) twins came to your clinic during routine examination ,you found great change behavior
between both of them this due to
a)hereditary
b)environment
c)maturation++++

184) radioliucent lesion with scalloped border above inferior alveolar canal between roots of
mandibular molars, this lesion is
a)solitary cyst
b)anyresmal bone cyst
c) traumatic bone cyst / simple bone cyst ++++

185) the use of low speed hand piece in removal of soft caries in child is better than high speed:
a) less vibration
b) less pulp exposure++++
c) better than high speed

186) gingival conditions occur in young adult has good oral hygiene was weakened:
a) ANUG++++
b) desqumative gigivitis
c) periodontitis
d) gigivitis

187) healing after scaling :


a) long junctional epithial ++++
b) new attachement

188) patient 6 years old came to your clinic after he had bicycle accident one day after,with
swelling at his lower lip, clinical examination shows ,teeth didn’t hurt, what you will do
a)x-ray ++++
b)endo for lower incisors
c)pulpotomy

189) child 3 years old came to clinic after fallen on his chin, you found that the primary incisor
entered the follicle for the permanant inicir what you will do
a)surgical removal of the follicle
b)leave it
c)surgicall removal of the primary incisors++++

Page | 24
190) Slow growing cyst at the site of 26,27,28 28 is missing
I don’t remember the answers but mostly <dentigerous cyst>

191) Dr.black (GV black) periodontal instrument classification:


study what the number represent in the instrument formula >>>>>

192) HBV disinfection: (enough with intermediate disinfection)


a) iodophores+++
b) dettol / 100% ethyl alcohol (all alcohols are not recommended)
c) hypcloride, ethyl
d) a+b

193) Dylantin (phynotoin) don’t give with :


b)azoles
c)metronidazole++++
d) all of the above

194) Pregnant 25 years, bleeding on probing, location on papilla of anterior area of the maxilla,
isolated:
a)giant cell granuloma
b) pyogenic granuloma (pregnancy epulis)++++
c) giant cell granuloma

195) Porcelain, highly esthetic , anterior maxilla area, we choose:


a) Dicor
b) in ceram++++
c)impress

196) the highest strength in porcelain:


a) ZR (zircon) reinforced in ceram++++

197) To check TMJ range of movement:


a) cranial imagery
b) arthrography++++ (source : oxford dental hand book)
c) traditional tomography
d) computerized tomography

198) Amalgam pain after restoration due to:


a) phase 2 gamma
b) phase 1 gamma
c) zinc containing alloy++++
d) admix alloy

Page | 25
199) an adult had an accident, maxillary central incisors intruded, lip is painful with superficial
wound what is the trauma’s classification:
a) luxation
b) sublaxation
c) laceration
d) abrasion
e) contusion.

200) Three year old pt, has anodontia (no teeth at all), what would you do:
a) full denture ++++
b) implant
c) space maitainer
d) no intervention

201) when sharpening instruments, unmounted stone has an advantage over mounted stone:
a) doesn’t change the shape of the cutting edge
b) easier to sterilize
c) eats less of the blade++++
remember: we use water with synthetic stones, and oil with natural stones

202) for root caries (class V) what’s the best restorative material:
a)composite resin
b)G I ++++
c)Amalgam

203) after bleaching a tooth, we want to restore the tooth with composite resin, we don’t want
to compromise the bonding, we wait for:
a) 24 hours
b) a week ++++ (ideal from 2-3 weeks : source “bleaching tech in resto. dentistry)
c) choose a different material

204) Rubber dam is important because it:


a) improves safety++++
b) looks scientific
c) improves suction.

205) Yellow tooth after trauma for primary dentition: • Remember that gray color
a) hemorrhage of the pulp necrotic pulp means : pulp necrosis in children.
b) calcifying degeneration of the pulp++++ • Red color means : haemorrhage
of the pulp

206) Swelling in the side of the face after inferior nerve block injection , you injected in:
-Facial artery’s branches or Parotid gland

Page | 26
207) The most superior Cold test for pulp:
a) ethyl chloride
b) ice block++++ (cause it’s localized to one tooth only).
c) cold spray

208) study Ceramic crown preparation and depth of preparation


ceramic thickness: 1 - 1.5mm
metal thickness: 0.5mm
all ceramic thickness: 1.5- 2.0mm

209) Pregnant woman, prophylactic treatment:


a) antibiotic++++
b) Corticosteroid
c) GA, none
d) None++++

210) Pt wants to extract his tooth but the diagnosis is RCT, he can’t afford the RCT, what’s you
action:
a) Ask him to go to another dentist
b) Give the advantage of keeping his tooth and how important it is and record his decision in his
chart treat him as he wishes++++
c) Extract exactly

211) Root canal pathoses mostly :


a) anaerobic obligated ++++ (1ry lesion)
b) aerobic
c) mixed

212) To take out a broken periodontal instrument from the gingival sulcus:
a) Schwartz Periotrever ++++

213) when take x ray in upperr premolar to locate lingual root using mesial shif it will apear
a) distal
b) mesial++++ Don’t forget that:
• SLOB rule means
c) buccal
same lingual < > opposite buccal.
d) lingual

214) tongue develope from:


a) mandibular arch &toburclum impar
b) 1st bronchial arch

Page | 27
215) endocrine and exocrine gland is :
a) pancreas++++
b) pittury gland
c) thyroid g
d) salivary g
e) sweat g

216) which is the most likely cause of periodontal cyst:


a) cell rest of malsze
b) cell rest of serss
c) cell of hertwig sheath ++++

217) perforation during endo preparation what is the most surface of distal root of lower molar
will have tendency of perforation:
a) Mesial Surface++++
b) Distal
c) Bucc
d) Ling

218) isolated pocket in :


-vertical root fracture++++
-palato gingival groove
-endo origine lesion
-all

219) crown and root perforation:


1/ respond to MTA
2/ use matrix with hydroxiapatite and seal with G I
3/1&2++++
4/root canal filling

220) 3rd generation of apexo locator:


1/ use with all pt
2/ need more research
3/increase chair time
4/ decrease radiographic film need++++

221) acceptable theory for dental pain :


1/hydrodynamic++++
2/fluid movement
3/ direct transduction

Page | 28
222) tracing of GP used for:
1/source of oeriapical pathosis++++
2/acute periapical periodontitis
3/ periodental abscess
4/ none

223) occlusal splint device:


1/ used during increase vertical dimension++++
2/allative muscle of mastication
3/ occlusal plane CR/CO
4/ALL

224) silane coupling agent:


1/ used with porcelain to enhance wetability of bonding++++
2/ used with tooth and porcaline

225) when increase vertical dimansion you have to:


1/ increase minimal need
2/construct anterior teeth first thenposterior teeth
4/ use provisional crown for 2 months
5/all ++++

226) Arrange the steps


answer : [ca(OH)2 placing > varnish > base > amalgam] .

227) Radiolucent structure arround the roots of mandibular vital premolars


-The mental foramen++++

228) Expensile radioopaque structure at canine/ premolar area


-Torus mandibularis++++

229) rigid palatal strap major connector the material of construction is


-co-cr++++
-Gold ti
-gold
-wrought wire

230) x-ray periapical for immature tooth is


-generally conculosive
-simply inconculosive++++
-should be compered with antermere

Page | 29
231) prophylactic antibiotic needed in
-anathesia not interaligamentary
-suture removal
-rotuine tooth brushing
-orthodontic band++++

232) >Soft palate falls abrubtly facilitates recording post dam.


>Soft palate falls gradually makes recording post dam difficult.
a) two statement true.
b) two false.++++
c) first true ,second false.
d) first false,second true.

233) prograssion of intial caries to cavatation takes 18 month this based on


a) streptococcus mutans inatiate caries++++
b) lactobacillia progress caries

234) caries progress in childern more than adult due to


a) differance in pH
b) generalized dentin scalerosing by age++++
c) increasing in organic content of tubular dentin by age

235) post length increasing will


a) increase retention ++++
b) increase resistant
c) increase strength of restoration

236) the following is true about periapical lesions:


a) When the source of irritation is removed healing in periapical area is good++++

237) osteogensis during endodontic surgery / root resectioning is aimed to prevent


a) fibrous in growth++++
Note:
b) growth factor
Root resection / Endodontic surgery = Apicectomy
c) formation of blood

238) 60 YEARS old patient need to make complete denture with thick labial freanum with wide
base the operation
a) vestibuloplasty++++
b) z-plasty (if narrow base frenum)
c) subperiostum incision
d) deepmucoperiostum incision

Page | 30
239) child patient take sedation before appointment and present with physical vault what should
dentist do
a) constious sedation
b) redo sedation ++++
c) tie with baboose board
d) tie in unit with bandage ++

240) class iii crown fracture in child patient the type of pontic
a) ovate ++++
b) egg shaped
c) hygienic
d) ridge lap

241) lesion at junction between hard and soft palate and surrounded with psudoepithelium
a) hyperplasia in salivary gland
b) necrotizing sialometaplasia++++

242) conditioning material of resected root


a) citric acid ++++
b) phosphoric acid
c) H2O2
d) EDTA

243) long tooth crown is managed by all of the following except


a) easily intruded orthodontically++++

244) the needle holder used in suturing of lower third molar


a) curved haemostat++++
b) allis forceps
c) Adson forceps

245) the structure of dye material is


a) propylene glycoal base++++

246) principle of elevator use of all the following except


a) wheel and axle++++

247) protocol of sterilization


a) intial cleaning - inspection - clean - steralization - storage++++
b) inspection-clean-sterilization-store

Page | 31
248) hand instrument which we used to make internal angles retentive grooves and preparation
of cavity walls in the cavity is
a) angle former++++
b) chisel
c) file
d) enamel hatched

249) amount of daily wear of amalgam ingested in the body


a) 3-4 Microgram/ day
b) 1-3 microgram / day++++

250) amalgam keep under


.fixer+++ (it should be epired diluted fixer solution).
.developer
.water

251) arrange the steps of cleft palate management


a) measures to adjust speech
b) establish way for nursing and feeding
c) cosmetic closure
d) prevent collapse of two halves
Answer : b > d > a > c
Note:
252) instrument used to handle the needle Addson Forceps <> soft tissue
a) curved haemostat++++ (or normal haemostat or needle holder) forceps (toothed / soft) to hold soft
b) addison forceps tissue , flaps …etc.
c) allis forceps Allis Forceps <> to hold intact or
ruptured cyst during removal & to
253) root canal irrigant used to kill E.feacalis remove epulis fissuratum.
a) NaoH
b) MTA++++
c) saline

254) oral surgeon put his finger on the nose of the patient and the patient asked to blow this
done to check
a) anterior extention of posterior palatal seal
b) lateral extension of posterior palatal seal
c) posterior extension of posterior palatal seal++++
d) glandular opening

255) patient with new amalgam restoration usually complains of pain with
a) cold++++
b) galvanic
c) hot

Page | 32
256) testing a tooth with porcelain fused to metal with
a) cold test
b) cold and hot
c) cold
d) cold and hot
e) cold with rubber dam++++

257) cost effective method to prevent dental caries


a) water fluoridation++++
b) flouridated tooth past

258) antifungal drug


a) nystatin++++

259) sharping of hand instrument mounted air driven better than unmounted due to
a) fine grift++++
b) sterilization
c) ability to curve instrument

260) die ditching means


a) carving apical to finish line++++
b)carving coronal to finish line
c) mark finish line with pen

261) instrument which we use to make groove in the wax is


a) carver
Note:
262) hot oven relative to to autoclave
a) the same time • Hot Dry Oven : 160° <> 2 hours
b) slightly higher time 200° <> 1 hour.
c) considerable higher time++++ • Autoclave : 127° <> 15-25 minutes
d) lower time 134° <> 3-5 minutes.

263) bronchial asthma epinephrine concentration subcutaneously


a) 1/1000++++
b) 1/10000
c) 1/100000

264) patient with leukemia Neutrophilic Count (ANC) is 1700 what oral surgeon should do
a) go on the maneuver
b) postpone another day Note:
c) work with prophylactic antibiotic ++++ Normal ANC is 1400 – 1600.
d) platelets transfusion

Page | 33
265) pain of short duration with hot and cold
a) dentin sensitivity ++++
b) irreversible pulpitis
c) chronic pulpitis
d) apical periodontitis

266) cyst in x- ray


a) radiolucent with bone expansion++++
b) radiolucent with bone resorption

267) pt. presented after insetion of complete denture complaining of dysphagia and ulcers what
is the cause of dysphagia
a) over extended ++++
b) over postdammed
c) under extended
d) under postdammed

268) color of normal gingiva in interplay between :


a) Keratin, b. vessels, melanin and epithelial thickness ++++

269) To select the shade of a tooth from a shade guide:


a) Under light
b) Dry tooth
c) None of above++++

270) black pt. came seeking for white teeth what should dentist do:
a) Show photos of other pt’s ++++
b) Do not show white teeth
c) Tell him firmly that these teeth are good

271) progression of initial caries to cavatation takes 18 month this based on


a) strepto.mutans inatiate caries++++
b) lactobacillia progress caries

272) the vertical fracture of the tooth detected by


a) periodontal pocket (not conclusive)
b) radiographically++++ (appears as a J-shaped lesion along the root affected)
c) vertical percussion (not conclusive)

273) cost effective method to prevent dental caries


.water fluoridation++++
.flouridated tooth paste

Page | 34
274) Amalgam restoration and there is also gold restoration in the mouth what should dentist do?
a) change restoration ++++
b) put separating medium
c) wait
d) put varnish

275) the temperature of melting gutta percha


a) 70 -160++++

276) for preparation reduction for functional cusp is :


a) 1.5mm
b) 2 mm++++

277) Pt came after 24 month of tooth replantation which had ankylosis with no root resorption
it most likely to develop root resorption in:
a) reduce greatly ++++
b) increase
c) after 2 years
d) after 4 years

278) Child with vesicle on the hard palate with history of malaise for 3 days what is the
possible diagnosis:
a) herpes simplex ++++ (OR Herpes Zoster)
b) erythema multiform

279) 5 years old pt had extraction of the lower primary molar & he had fracture of the apex of
the tooth what is the best treatment :
a) aggressive remove
b) visualization & remove
c) visualization & leave ++++

280) 7 y/o boy came to the clinic in the right maxillary central incisor with large pulp exposure:
a) pulpectomy with Ca(OH)2
b) pulptomy with Ca(OH)2 ++++ • Pulpotomy/ Apexogenesis: in case of healthy pulp tiss. / small
exposure.
c) Direct pulp capping
• Pulpectomy: in case of infected pulp / Large exposure.
d) leave it • Apexification: in case of open apex with infected / dead pulp.

Page | 35
281) 30 years old pt came to the clinic with brownish discoloration of all his teeth (intrinsic
discoloration) & yellowish in U/V light the most likely cause is:
a) flourosis
b) tetracycline ++++
c) amelogensis imperfecta
d) dentogensis imperfecta

282) The most common professional use of flourid in peado are:


a) Acidulated phosphate flouride (APF)++++ APF : Na2F / Stannous Flouride in
an acidic medium of Phosph. Acid.
PH : 3/4
283) Pt came to the clinic complaining from soreness in the tongue sore throat the diagnosis is:
a) burning mouth syndrome ++++
b) geographical tongue
c) fissure tongue

284) Mucoceles the best ttt is:


a) Excisional ++++
b) leave it
c) marspuilization (For Ranula Only)
d) cauterization

285) wax properties are:


a) expansion
b) internal stress++++

286) old pt came to replace all old amalgam filling he had sever occlusal attriation the best
replacement is:
a) composite
b) amalgam
c) cast metal restoration
d) full crowns++++

287) Teenager boy with occlusal wear the best treatment is:
a) remove the occlusal
b) teeth capping ++++
c) restoration

Page | 36
288) baby born without which bacteria:
a) Streptococcus mutans++++

289) avulsed tooth:


a) splint 7-14 day ++++
b) 3- 14 weeks

290) difference b/w gracy curette & universal curette:


a) Cross section in Gracey is semicircular and in universal is triangular.
b) Gracey has one cutting edge while universal has two cutting edges.
c) Gracey has a certain site to work on per instrument, while universal can be applied any where
in the dentition
d) Gracey has offset of 60°, while universal has offset of 90°
e) All of the above.
f) b, c, d++++

291) Acute periapical abscess & acute periodontal abscess are differentiated by:
a) vitality test ++++

292) Mobility of the midface bilateral in frontozygomatic suture:


a) lefort III ++++

293) Etching dentine is :


a) 15 sec ++++

294) Polishing bur have:


a) more than 12 blade ++++

295) rampant caries in adult in anterior teeth restore by:


a) GIC ++++

296) best provisional restoration coverage for anterior teeth is:


a) tooth colored poly carbonate crown++++

297) disinfection of gutta p :


a) chemical solution ++++ (chlorohexidine &/or NaOCl)

Page | 37
298) during post removal the first thing to do is:
a) remove the G.P
b) remove all the old restoration & undermined enamel & caries ++++
c) insertion of post immediately

299) Young pt complete the orthodontic treatment. Upon removal we found


white spot in ant. Teeth. how to restore
a) microscalar then fluoride application++++ (Prevention)
b) composite veneer.

300) Epliptic pt should not take which drug


a) aspirin
b) metronidazole ++++

301) Pt came with small trauma of the lower lip & discoloration of his tooth what is the name
of this condition
a) abrasion & subluxation
b) laceration & subluxation ++++

302) Young pt came without any complain during routine X ray appear between the two lower
molar lesion diameter about 2mm & extend laterally with irreg. shape what’s the type of cyst
a) dentigerous cyst++++
b) apical cyst
c) radicular cyst

303) Pt had generalized lymphoadenopathy. the diagnosis is?


a) infection +
b) HIV ++++

304) Nitrous oxide effect to?


a) vit A
b) vit B6
c) vit B12++++

305) Defect factor VIII is which kind of hemophilia?


a) Hemophilia A++++ (defect factor 9 : hemophilia B)

Page | 38
306) It has been proven that amalgam restoration has the following characteristics
1) Micro leakage decrease with aging of the amalgam
2) It is the least technique sensitive of all current direct restoration
3) High dimensional changes
Choose the appropriate answer:
a) 1,2 & 3
b) 1 & 3
c) 1 & 2++++
d) 2 only

307) To distinguish periapical and periodontal abscess we use


a) X-ray examination
b) Clinical examination
c) Vitality of the pulp++++

308) Single rooted anterior teeth needs endodontic treatment which is best treated by:
a) Casted post & core. ++++
b) Preformed/ Prefabricated post & composite.
c) Preformed post & amalgam.
d) Composite post & core.

309) Squamos cell carcinoma is a malignant from


a)skin
b)mucous membrane
c)glandular tissue
d)mucous epithelial membrane++++

310) The antibiotic of choice in endodontics


a) Metronidazoles
b) Penicillin &…. ++++
c) Tetracycline
4)Erythromycins

311) Selection of shade for composite is done:


To rest the eyes, NEVER YELLOW, but
a) before preparation++++ high contrast colour : Ex. Blue/green “towel
b) we must rest the eye by looking to a yellow color. of the patient”
c) we must look to the tooth only 5 sec++++

Page | 39
312) Commonly, after placement of amalgam restoration pt. complain from pain when eating
a) Hot
b) Cold++++
c) sweet

313) Irrigation solution for RCT ,when there is infection and drainng from the canal is
a) Sodium hypochlorite
b) Iodine potassium
c) sodium hypochlorite and iodine potassium ++++

314) Time of curing of dentin


a) 10 secs
b) 15 secs
c) 30 secs ++++
d) 60 secs

315) Squamous cell carcinoma is


a) multifactorial ++++

316) If tooth or root is pushed during surgical extraction into maxillary sinus
a) Leave it and inform the patient
b) Remove it as soon as possible ++++
c) Follow the patient for 3 months
d) None of the above

317) for discharged sharp instrument (blades, needle tips, wedges, …etc) put in :
a) dicharged paper basket
b) disgned sharp instrument special container++++
c) disinfectant in auto glave then throw
d) put it in multifoil

318) what is the time bet. the first onset of HIV virus and the appearance of acute symptoms :
a) 1-5 years
b) 9-11 years++++
c) no specific time is known
d) above 10 years

Page | 40
319) why the moisture heat sterilization is better than dry heat sterilization
a) makes the instruments less rusty and blunt
b) needs more time and affects the proteins of the cell membrane
c) needs less time and affects the proteins of the cell membrane++++

320) Periodontal ligament fibers in the middle third of the root is


a) Oblique++++ (most of the PDL fibers)
b) Horizontal
c) Trans-septal

321) Autoclave relative to 100f dry oven


a)the same time
b)slightly higher time
c)considerable higher time
d)less time++++

322) pt came after few days from insertion complete denture ,there is no complain ,except the
sound specially letter he say it like (v), the reason is ?
a)the anterior upper teeth are upward from the lip line++++
b)the anterior upper teeth forward from the lip line

323) father for child 12 year patient, asked you about the age for the amalgam restoration of his
child, you tell him:
a)2 years
b)9 years
c)2 decades++++
d)all life

324) pt have unilateral fracture of left the condyle the mandible will
a)deviate to the left side++++
b)deviate to the right side
c)no deviation

325) pt have a complete denture came to the clinic ,tell you no complaint in the talking ,or in
the chewing ,but when you exam him ,you see the upper lip like too long ,deficient in the
margins of the lip, reason is
a)deficiency in the vertical dimensional
b)anterior upper teeth are short++++
c)deficiect in vit B

Page | 41
326) pt with complete denture come to your clinic ,complaining from his dry mouth ,the proper
medicine is
a)anti diabetic medicine
b)anticordial ++++
c)steroids

327) reduction in the incisal edge for metal porcelain crown is


a)1mm
b)1.5mm
c)2mm++++
d)4mm

328) for 3 years child, the toothpaste with fluoride for him is
a)recommend++++
b)not useful
c)toxicity
d)must not be used

329) 4 year child come to restore his lower, first molar with destruction in 3to 4 surfaces ,the
best restoration
a)amalgam
b)full porcelain crown
c)full metal crown++++
d)extraction

330) female come need to endodontics for central incisal ,and have compsite restorations in the
mesial and distal walls ,and have attrition in the incisal edge the best restoration?
a)jacket crown
b)full crown++++
c)metal cown

331) child 10 years came with trauma on the center incisor from year ago ,and have dis
coloring on it , in the examination ,no vitality in this tooth ,and in the x ray there is fracture from
the edge of the incisal to the pulp ,and wide open apex the best treatment ?
a)calcification++++
b)RCT with gutta percha
c)extraction
d)capping

Page | 42
332) pt with hepatitis B, the best sterilization is
a)formaldahyde
b)detol+100%alkohol
c)iodophores ++++
d)…..

1)a&b ++++
2)a&b&c

333) the best core material in the anterior teeth is?


a)composite++++ (Casted P & C is better)
To increase Alginate setting time:
334) how can you alter the sitting time for alginate
a)alter ratio powder water++++ 1. Use cold water & spatula.
b)alter water ratio 2. Increase water <increase
water/powder ratio).
c)we can’t alter it
3. Increase speed of spatulation / mixing.
d)by accelerated addition

335) application of rubber dam in the endodontics


a) necessary
b)patient comfort
c)established rule ++++
d)extra cost

336) amalgam restoration with post & core in the posterior teeth decide by
a)canal curvature
b)canal length and diameter
c)Amount of crown destruction++++

337) Female patient came to your clinic with continous severe pain related to 1st maxillary
molar. After examination dentist diagnose the tooth is carious and has irreversible pulpitis. He
decides to do RCT. After enough time for anaesthisation, the patient won’t allow the dentist to
touch the tooth due to severe pain. Dentist should:
a) give another appointment to the patient with description of antibiotics.++++
b) Extraction.
c)Intra-pulpal anaesthia.

Page | 43
338) 32 years old patient came to your dental office, suffering from a bad odour and taste from
his mouth. By examination patient has an anterior mandibular 3 unit bridge that bubbles upon
applying water spray and slight pressure. Cause:
a) broken abutment.
b) Food impaction underneath the pontic.
c) separation between the abutment and the retainer.++++ (dissolving of cement / microleakage)

339) What’s the name of the Device used to measure the Vertical Dimension
a) caliper++++
b) face bow

340) Patient complaining from pain in the floor of the mouth (beneath the lower jaw) your
diagnosis is related to the salivary glands, what’s the best x-ray to help you:
a)panoramic
b)occlusal
c)sialograph++++ (Specialized radiograph for the Salivary gland disorders)

341) what is the foreceps to remove epulis fissaratum:


a) allis++++
b) adison

342) distal surface for first upper premolar ,contact with the neighbor teeth :
a)in the middle with buccal vastness wider than lingual one
b)in the middle with lingual vastness wider than bucccaly one++++

343) preparation for labial surface in one plane in the preparation for metal crown is:
a)more retentive
b) less retentive ++++
c)…..
d)…..

344) the movement of polymorphic cells in the gaps of intracellular to the blood capillary
outside it called:
a)porosity
b)slinking
c) diapedesis++++ (source Wikipedia)

Page | 44
345) Patient suffering from pain in the area of the mandibular molars with parasthesia in the
lower lip. By clinical and radiographic examination your diagnosis:
a) Acute osteomylitis++++

346) What is the dominant type of fibers found in Cementum:


a) longitudinal
b) Circular
c) Sharpey’s fibers.++++

347) What is the main function of impression tray holes :


a)Fixing the Impression material++++

348) A Tailor is presented to your dental office, what’s the most common feature to be found in
his teeth upon examination :
a)Attrition
b) abrasion ++++
c) Erosion
d) Abfarcation

349) what’s the first sign of Syncope


a) Paleness++++
b) nose bleeding (epistaxis)
c) Miosis

350) upon giving a lower mandible anaesthia, you notice the patient’s eye, cheek corner of the
lip are uncontrolled , what’s the reason :
a)Parasthia of the Facial Nerve +++

351) What’s the reason of the wax shrinkage upon fabrication of the bridge/crown :

352) Upon opening an incision in a periapical abscess in a lower 1st molar, you open :
a) The most bottom of the abscess ++++ (for continous evacuation of pus through the drain
b) The most necrotic part of the abscess ++++ (the most fluctuant part of the abscess)
c) Extra oral

Page | 45
353) Patient came to your clinic complaining of pain, upon examination you can’t find a clue.
What’s the next logical step to do in investigation
a) Panoramic x-ray++++
b) CT Scan
c) MRI
d) Regular tomography

354) What’s the best implant type allowing Osseointegration:


a) Root-form Endosseous implant.++++

379) what medical condition should prevent the dentist from practicing dentistry :
a) Diabetes
b) Hypertension
c) Influenza++++
d) Headache

380) where does the breakdown of Lidocaine occurs :


a) kidneys
b) Liver++++

381) patient complaining of Xerostomia & frequent going to the toilet at night
a) Diabetes Mellitus++++

382) what is the most factor encouraging dental caries :


a) Xerostomia++++
b) Hypocalcification
c) Smoking

383) what is the type of sterizliation applied on ligation/fixation wires


a) Autoclave++++

384) child suffering a trauma resulting in a complete avulsion. The more successful is when :
a) immediate++++
b) after 24 hours

385) a child at dentation age is suffering from :


a) diahrrea
b) sleep disorders
c) increased salivation++++

Page | 46
386) which of the following materials is NOT a hemostatic agent :
a) Oxidized cellulose
b) Gelvon
c) Zinc Oxide++++

387) time of PT, PTT :


a)11-15 seconds , 25-40 seconds++++

388) For cavity class II amalgam restoration in a second maxillary premolar, the best matrix to be
used:
a) Tofflemire matrix++++
b) Mylar matrix
c) Gold matrix
d) Celluloid strips

389) A completely edentoulus patient, the dentist delivers a denture in the 1st day normally, 2nd day
the patient returns unable to wear the denture again, the cause is :
a) Lack of Skill of the patient ++++
b) Lack of Frenum areas of the Complete denture.

390) for a newly erupted tooth, the most bacteria found around the tooth is :
a) Streptococcus mutans++++
b) Streptococcus salivaris

391) generalized gray discoloration in a 28 years old patient’s teeth, with blue sclera and an
enlarged pulp chambers and short roots, and multiple fractures in Enamel… the diagnosis is :
a) Dentinogenesis Imperfecta++++
b) Amelogenesis Imperfecta

392) For a patient that is on a corticosteroid therapy, upon oral surgery, the patient is given :
a) 100 - 200 mg hydrocortisone++++
b) 400 - 600 mg prednisolone

393) an 8 years old child, suffered a trauma at the TMJ region as enfant. Complaining now from
limitation in movement of the mandible. Diagnosis is :
a) Sub luxation
b) Ankylosis++++

Page | 47
394) Removal of Undermined Enamel in Class II cavity is done by :
a) Chisel ++++
b) Angle former
c) Excavator

395) The least effective method to kill the HIV is through


a) NaOCl
b) Autoclave
c) Chemiclave
d) Ultraviolet chamber++++

396) instrument used to catch the flap (soft tissue) when we do impaction in lower third molar :
a) Addison forceps++++
b) Allis forceps
c) Curved hemostat
d) Regular tweezer

397) The outline form of upper maxillary molar access opening is Triangular, The base of this
triangle is directed toward :
a) Buccal++++
b) Palatal
c) Mesial
d) Distal

398) patient suffering from a submandibular gland abscess, dentist made a stab incision and is
fixing a rubber drain to evacuate the pus, the drain is sutured to :
a) Intra-oral
b) From angle of the mandible++++
c) Between myloid muscle and…..

399) The best material for taking full crown veneers impression is :
a) Poly-sulphide
b) Poly-ether
c) Irreversible
d) Poly vinyl siloxane (Additional silicone) ++++

Page | 48
400) Patient came to your office suffering from paing in upper second premolar… after x-ray you
found the tooth had a failure in RCT obturated with Silver cones. What is the most suitable
method to remove the silver cone :
a) Hatchet ‫)بحسب ما جاء في فيديو لدكتور رادل )استشاري عالج أعصاب‬
b) Steiglitz pliers++++ ‫ أفضل طريقة لنزع السيلفر بوينت ھي عن طريق‬، (‫أمريكي‬
(.‫الستايجلتز بالير ثم اآللترا سونيك ثم اإلتش فايل‬
c) Ultrasonic tips+++
d) H files++

401) what is the concept of Pro-taper system :


a) Step down tech. Source: http://www.dental-
b) Step back tech. update.co.uk/articles/35/3502110.pdf
c) Crown down tech. ++++

402) after patient came to your clinic and gave an extended history and complain, what’s your next
step in treatment :
a) Clinical examination++++
b) Start the treatment
c) Radiographic examination

403) method of Detection of Cracked teeth :


a) Horizontal percussion
b) Vertical percussion
c) Electric pulp test
d) Transillumination / visible light test.++++

404) Patient suffering from a cracked enamel, his chief complain is pain on :
a) Hot stimuli
b) Cold stimuli
c) A & B++++
d) Electric test.

405) cause that master cone not reaching working length:


a)ledge
b)blockage by debris++++

406) Preparation of tooth for metal ceramic restoration should be done in:
a) two planes++++++
a) parallel to long axis

Page | 49
407) Pt presented with vehicle accident u suspect presence of bilateral condylar fracture what is
the best view to diagnose cond. Fracture:
a) occiptomenatal
a) reverse towne’s ++++
c) p/a
reverse towne for fracture of condylar neck &ramus areas (dental decks)

408) When extracting all max teeth the correct order is:
a) 87654321++++
b) 87542163
c) 12345678

409) Trauma caused fracture of the root at junc. Between middle and cervical third:
a) do endo for coronal part only
b) RCT for both
c) leave
d) extraction++++

410) when removing lower second molar :


a) occlusal plane perpendicular To the floor
b) buccolingual direction to dilate socket+++++
c) mesial then lingual

411) after u did RCT to your pt he came back to the clinic after few days with severe pain on
biting, you did x-ray and it revealed that the RCT filling is very good, but u saw radiopaqe,
thin (film like) spot on the lateral border of the root
what is the most probable diagnosis,sn
a) Accessory canal
b) vertical root canal fracture++++

412) What is the disavdantages of Mcspadden technique in obturation


ANSWER :
 Use of speed higher than recommended poor seal.
 Extrusion of the filling material.
 Fracture of thermocompactor.
 Gouging of the canal wall.
 Inability to use the technique in curved canals.
 Heat generation may lead to PDL damage, resorption and ankylosis..
 Voids in final filling
all of these are correct, either of them will come as one of the choices.(Source: different
textbooks)

Page | 50
413) Pt. presented after insertion of complete denture complaining of dysphagia and ulcers
what is the cause of dysphagia?
a) over extended++++
b) over postdammed
c) under extended
d) under postdammed

414) Trauma lead to fracture in the root between middle cervical apical third
a) poor prognosis++++
b) good prognosis

415) The best material for taking impression for full veneer crowns:
a) polyvinyl sialoxane (additional Silicone)++++

416) Upon opening an incision in a periapical abscess in a lower 1st molar, you open :
a) The most bottom of the abscess
b) The most necrotic part of the abscess
c) Extra oral

417) What’s the test used for HIV


‫إختبار تحليل السيدا المندرج تحت إختبار اليسا‬

418) Which type of burs is the least in heat generation:


a) diamond
b) carbide++++
c) titanium
*most heat generated in diamond burs
dental secrets: pg 200

419) Pt. wears complete denture for 10 years & now he has cancer in the floor of the mouth.
what is the first question that the dentist should ask:
a)does your denture is ill fitted
b) smoking+++++ (80% of the cancer of the floor of the mouth is caused by smoking)
c) Alcohol
d) does your denture impinge the o.mucosa++++ (traumatic cause)

Page | 51
420) Dental student using thermoplastsized g.p. what is the main problem he may face;
a) extrusion of g.p. from the canal++++
b) inability to fill the proper lengh++
c) failure to use maser cone at proper length
d) ledge

421) U shaped radiopacity in a panorama is :


a) Zygoma
b) Wall of the maxillary sinus
c) Zygomatic arch / process ++++

422) Doing cantilverers, we consider all of the following except:


a) small in all diameters
b) high yield strength
c) minimal contact
d) small occlosgingival length++++ (in abutments)

423) Doze corticosetriods


a) 100 -200 mg++++
b) 40 - 60 mg

424) Silica cement


a) first tooth color restoration
b) use permanent filling
c) 15% f
d) all ++++
e- a&c

425) Bacteria in root canal


a) mixed
b) anaerobes obligatory ++++
c) aerobes only

426) Secondary dentine occur due to


a) occlusal trauma
b) recurrent caries
c) attrition dentine
d) all of the above++++

Page | 52
427) All of these are ways to give L.A with less pain except:
a) give it slowly
b) stretch the muscle ++++
c) Topical anesthesia
d) the needle size over than 25 gauge

428) The following are indication of OUTPATIENT general anesthesia except


a) ASA categories 1 & 2
b) the very young child
c) cost increase
d) Patient admitted and discharge the same day

429) Flouridated toothpaste for 3 years child is recomended but under supervision in small pea-
sized amount
a)recommended++++
b)not useful
c)toxic
d) limited
Source :
http://www.fluorideinformation.com/guide_to_fluoride/toothpaste#When%20should%20m
y%20child%20start%20using%20fluoride%20toothpaste?

430) Class iii crown fracture in child patient the type of pontic
.ovate++++
.egg shaped
.hygienic
.ridge lap

431) What is the benefit of rinsing the mouth with water:


a) Plaque removal
b) calculus removal
c) washing the food debris++++

432) In restoring lost tooth, which is least important :


a. esthetic
b. pt demand++++
c. function
d. arch integrity and occlusal stability

Page | 53
433) Composite restoration follow-up after 2 years. It showed stained margin, this is due to:
a. Stress from polymerization shrinkage++++
b. Hydraulic destruction on bond (microleakage)

434) Which intra-canal medicament causes protein coagulation:


a. formacresol++++
b. NaOCl
c. wad

435) GIC compared to composite:


a) increased linear cof, of thermal expansion (actually GIC has less)
b) more wear resistant
c) less soluble
d) stiffer
e) less polymerization shrinkage++++

436) Buccal branch of facial nerve is:


a) Sensory
b) Motor ++++
c) Mixed

437) Lesion at junction between hard and soft palate and surrounded with psudoepithelium
hyperplasia in salivary gland
a) Necrotizing sialometaplasia++++

438) Pt. presented to u complain of click during open and close. Thers is no facial asymmetry
except when opening
What is the diagnosis:
1-internal derangement with reduction++++
2-internal derangement without reduction
3-reumatoid arthritis
Disc dislocation with reduction
‫ھو عبارة عن انزالق القرص المفصلي من مكانه لياخذ وضع غير وضعه الطبيعي فاثناء اغالق الفم يكون امام اللقمة ثم يتراجع‬
‫) اثناء الفتح واالغالق‬click(‫لياخذ مكانه على راس اللقمة اثناء االغالق وھذا يؤدي الى سماع صوت‬

Disc dislocation without reduction


‫يبدأ القرص المفصلي بعدم العودة للخلف ويبقى امام الناتى المفصلي ويصبح فتح الفم صعبا ومؤلما وقد يصبح المضغ شبه‬
‫ عند الفتح واالغالق‬click ‫مستحيل وعندھا اليصدر المفصل الفكي‬

Page | 54
439) An 8 years old child, suffered a trauma at the TMJ region as enfant. Complaining now
from limitation in movement of the mandible. Diagnosis is :
a) Sub luxation
b) Ankylosis++++

‫ ألن االنكيلولزس غالبا مايكون ناجما عن رض عند االطفال ترك لفترة طويلة ولم يعالج مما يؤدي الى تحدد‬Ankylosis ‫الجواب ھو‬
‫وثنائي الجانب‬bony ankylosis ‫ او احادي الجانب ام اذا كان‬fibrous ankylosis‫شديد في حركة الفتح واالغالق وخاصة اذا كان‬
‫فھنالك تحدد كامل في فتحة الفم‬
‫ ھنالك امكانية في فتح الفم واغالقه وغالبا يترافق مع لقمة مفصلية )مع او بدون القرص المفصلي( امام‬Sub luxation ‫ام في حالة‬
‫عن ال‬Sub luxation ‫الحدبة المفصلية ممايؤدي الى وضع القفل ولكن المريض يستطيع ان يرجع فكه في حالة انخالعه وھذا مايميز‬
.‫الحقيقي بحيث اليستطيع المريض ارجاع فكه لوحده‬luxation

440) The cement under MOD amalgam have this character:


a- high modulus of elasticity (stiffer)++++
b- low modulus of elasticity.
c- the high modulus of elasticity prevent bonding and decrease tensile strength.
d- both a &c

441) Dentist provided bleaching which also known as (home bleaching) contains
a- 35-50% hydrogen peroxide
b- 5-22%carbamid peroxide++++

442) During endo. Pt. is complaining of pain with percussion what u suspect?
a-apical periodontitis
b-secondary apical periodontitis++++
c-over instrumentation
d-over medication

443) Missing lower six and tilted 7


a- Uprighting of molar by orthodontics++
b- Proximal half crown++++
c- Telescope crown++
d- Non rigid connector++

444) To check TMJ range of movement:


a) cranial imagery
b) arthrography++++
c) traditional tomography
d) computerized tomography
Page | 55
Acyclovir dose for treatments of herps :
a) 200 mg / 5 times a day ++++
b) 200 mg / 4 times a day
c) 400 mg / 4 times a day
d) 800 mg / 4 times a day
in case of immunodeficiency double the dose to 400 mg

445) A female patient came to your clinic with dry lips and mouth and bilateral submandibular
oedema and ocular dryness. Diagnosis is :
a)Polymorphecadenoma
b) sialotitis++++
or : Sjögren's syndrome if present

446) How much subgingivally do you go with the band in class II restorations:
a) 0.5 – 1 mm++++
b) 1 – 2 mm
c) 2 – 3 mm

447) The compression / relaxation cycle of external cardiac compression should be repeated
a- 2 times / second
b- 60 times / minute ++++ (Source : text book)
c- 76 times / minute
d- 100 times / minute

448) One of the primary considerations in the treatment of fractures of the jaw is
a- to obtain and maintain proper occlusion++++
b- test teeth mobility
c- vitality
d- embedded foreign bodies

449) A child patient undergone pulpotomy in ur clinic in 1st primary molar. Next day the
patient returned with ulcer on the right side of the lip… your diagnosis is :
a) Apthosis
b) Zonal herpes
c) traumatic ulcer++++

Page | 56
450) Bitewing exam is used to diagnose EXCEPT:
1. Proximal caries.
2. Secondary caries.
3. Gingival status.
4. Periapical abscess.++++

451) We can use under the composite restoration:


1. Varnish.
2. Zinc oxide and eugenol.
3. Ca (OH)2.
4. Zinc phosphate cement.
• 1+2.
• 2+3.
• 3+4.++++
• 2+4.

452) Autoclaving technique is depending on:


a. Dry heat.
b. Steam heat.++++
c. Chemicals.

453) The inferior alveolar nerve is branch of:


1. Mandibular nerve – not divided.++++
2. Posterior mandibular alveolar nerve.
3. Anterior mandibular alveolar nerve.

454) Tooth with crown fraction under the gingiva and we want to use it, the treatment is:
1. Amalgam post & core under the gingival.++++
2. Extrusion orthodontic.++++
3. Restoration under the gingival.

455) A child (2 years) with caries in the incisors we call this caries:
1. Rampant caries.
2. Nursing caries.++++
3. Children caries.

456) Persons who are working in glass factories they have the disease:
1. Silicosis.++++
2. Asepsis.

Page | 57
457) Laser used in endodontic is:
1. Co2.
2. Nd (YAG).++++ (biolase/waterlase)
3. Led.

458) A patient 14 years with avulsed incisors 10 and 21 we can use a splint for:
a) 1 – 2 week.++++
b) 2 – 3 week.
c) 3 – 4 week.
d) 4 – 5 week.

459) The most frequent cause of porosity in a prcelain restoration is


a- moisture of contamination
b- excessive firng temperature
c- excessive condensatin of the porcelain
d - inadequate condensation of porcelain++++ (dental decks 441)

460) A compound fracture is characterized by


a-many small fragments
b -a star shaped appearance
c- an incomplete break in the bone
d- commuication with oral cavity++++

461) Which of the following types of base materials can be placed in contact with polymethyl
methaacrylate & not inhibit the polymerization of the resin
a) ZoE
b) GI cement
c) Zn phosphat cement
d) varnish
e) b&c++++

462) In the preparation of cavity class II for restoration with composite resin all cavo-surface
angles should be
a-well rounded
b- right angles
c- acute angles
d-obtuse angles
e- even

Page | 58
463) The most desirable finished surface composite resin can b provided by
a) white stones ++++ (or aluminum oxides if present )
b) hand instruments
c) carbide finishing burs
d) diamond finish burs
e) celluloid matrix band

464) Indirect composite inlay has the following advantages over the direct comp Except
a) efficient polymerization
b) good contact proximally
c) gingival seal
d) good retention++++

OR

465) indirect composite inlay over come the direct composite by


1/ insusffition polymerization
2/good contact proximaly
3/ gingival seal
4/ good retention
1/ 1-2-4
2/ 1-2-3 ++++
3/ 4-3

466) After class V GI restoration removal of a thin flush of GI is done by


a) scaler or knife immediately
b) finishing stone immediately
c) scaler or knife later
d) finishing stone later
e) a&b
f) a&d ++++

467) In class V composite restoration. A layer of bonding agent is applied


a) following removal of cement then cured++++
b) following removal of cement then not cured
c) cured then remove cement
d) none of the above

Page | 59
468) Marginal deterioration of Amalgam restoration may be due to
a) no enough bulk of dentin ( undermined enamel)
b) corrosion
c) over carving
d) improper manipulation of amalgam
e) b, c & d ++++
f) all of the above

469) A restoration of anterior teeth with RCT, abraded incisal edge & small M&D caries is
done by
a) ceramo-metal crown ++++
b) composite
c) laminated veneer
d) none of the above

470) Powder for GI cement contains


a) SiO2, Al2o3, CaF ++++
b) SiO2,ZnO, barium sulphate
c) none of the above

471) The following cavity bases r moisture sensetive


a) Znpolycarboxylate
b) Zn phosphate
c) GI cem ++++
d) ZnO eugenol
e) a & c

472) minimal facial reduction when preparing for veneers:


a) 0.3 mm
b) 0.3-0.5 mm++++
c) 1-1.5 mm

473) Amount of reduction of functional cusp when preparing for onlay


a) 1
b) 1.5 mm++++ (dental decks) Porcelain / Gold Onlay : 1.5 mm
c) 2 mm
Amalgam Onlay : 2.5 – 3 mm
d) 2.5 mm

Page | 60
474) Fractured tooth to alveolar crest, what's the best way to produce ferrule effect?
a) restore with amalgam sub-gingivaly++++
b) crown lengthening+ (DisAdv : increase crown/root ratio)
c) extrusion with orthodontics+ (DisAdv : extra cost, extra time)
(source : http://www.dental-update.co.uk/articles/35/3504222.pdf)

475) firm fixed neck node are most apparent to be detected in association with :
a) amelo blastoma
b) basal cell carcinoma
c) sequamous cell carcinoma++++

476) Cracked tooth syndrome is best diagnosed by?


a) radiograph
b) subjective symptoms and horizontal percussion
c) palpation and vertical percussion ++++
d) pulp testing

477) Patient comes to you with edematous gingiva, inflamed, loss of gingival contour and
recession, what's the best tooth brushing technique?
a) modified bass
b) modified stillman
c) charter++++
d) scrub

478) The best method to protect teeth that underwent bicuspidization procedure from fracture?
a) full crown++++
b) splint with composite
c) orthodontic splint

479) Verrucous carcinoma is:


a) malignant++++
b) non-malignant
c) hayperplastic
d) none of the above

480) Geographic tongue is seen in:


a) Diabetes mellitus
b) Iron deficiency++++
c) none of the above

Page | 61
481) What is the concept of Pro-taper system:
a) Step down tech.
b) Step back tech.
c) crown down++++

482) Fluoride which we use in the clinic doesn’t cause fluorosis because
a) It’s not the same fluoride that causes fluorosis
b) Teeth already calcified ++++
c) Calcium in the mouth counter
d) Saliva wash it out

483) Nausea is a complaint that a new denture wearer might encounter. It may result from
a) Thick posterior border
b) Denture under extended
c) Denture slightly overextended ++++
d) a & b both are correct

484) Cause of angular chelitis:


a) loss vertical dimension pt have complete denture++++ (due to Drolling of saliva)
b) iron deficiency anaemia++++ (one of the signs)
(dental decks page 1334)

485) After etch enamel and bond it with 5th generation the strength of
a) 5-10Mp
b) 25Mp++++
c) 30Mp
d)100Mp

486) Perio endo lesion the primary treatment:


a) endo treatment++++
b) deep scaling and root planning
Source (Dental decks page: 216)

487) Provisional luting cement:


a) prevent restoration from dislodgement++
b) Sealing++++

Page | 62
488) A removable partial denture patient, Class II Kennedy classification. The last tooth on the
left side is the 2nd premolar which has a distal caries. What’s the type of the clasp you will use
for this premolar
a) gingivaly approaching clasp++++
b) ring clasp++

489) Electric pulp tester on the adults is not accurate because :


a) Late appearance of Fibers A++++
b) Late appearance of Fibers C
c) Early appearance of fibers A
d) Early appearance of fibers C

490) A patient complaining from a severe oedema in the lower jaw that increases in size upon
eating, Diagnosis is :
a) salivary gland++++ (submandibular sal. gl.)

491) A patient that had a class II amalgam restoration, next day he returns complaining of
discomfort at the site of the restoration, radiographically an Overhanging amalgam is present.
This is due to:
a) lack of matrix usage. ++++
b) no burnishing for amalgam

492) Contents of the Anaesthia carpule:


a)Lidocaine + epinephrine + Ringer’s liquid ++++
b)Lidocaine + epinephrine + distilled water
c) Lidocaine + epinephrine only

493) thermo mech. tech of obturation is :


a) thermafil
b) obtura
c) ultrafil
d) mcspadden++++ (source : Endodontic obturation materials)

494) endomethasone is aroot canal sealer that:


a- dissolv in flud so it weaken the root canal filling
b- very toxic contain formaldehyde
c- contain corticosteroids
d- all the above ++++
(source : http://www.endoexperience.com/filecabinet/Clinical%20Endodontics/Obturation/Sealers/SEALERS.pdf)

Page | 63
495) A patient that wasn’t anaesthized well in his 1st visit, next day he returns with a limited
mouth opening (trismus). He must be anaesthized, what’s the technique to be used :
a) William’s technique
b) Bercher’s technique++++
‫طريقة التخدير المستطبة في حال لم يستطيع المريض فتح فمه )الضزز‬-Trismus)‫ ھي طريقة‬Bercher ‫وتستخدم لتخدير العصب‬
‫عبد‬.‫الماضغ)الفرع الحركي من السني السفلي(للوصول الى ارتخاء الفك السفلي وھذه الطريقة التي اعرفھا وھي مذكورة في كتاب د‬
‫الكريم خليل )التخدير الموضعي لجراحة الفم والوجه والفكين الجزء الثاني( وھناك طرق اخرى ولكن ھذه اشھرھا‬

496) young pt with amalgam after week come with pain due to
a) gamma 1
b) gamma2
c) Zinc containing alloy++++

497) 55 old pt wear upper maxillary complete denture since 3 year against 6 lower ant teeth,
what’s the clinical picture present :
a) flabby lower post ridge of mandible
b) osseous resorption in maxillary ant area++++
c) severe destruction in post area of the mandible

498) isolation period of chickenpox should be


a) after appear of rash by week
b) until vesicles become crusted++++
c) until carter stage is last

499) Fluoride use for children


a) fluroide thio sulphate
b) Acidulated phosph. fluoride <stannous fluoride / Na2F>++++

500) scale to measure marginal deterioration


a) mahler scale++++
b) color analogue scale

501) 10 years pt come with necrotic pulp in upper central with root apex not close yet best
treatment
a) calcium hydroxide
b) calcifuic barrier ++++
c) apexfication with gutta percha filling
d) gutta percha filling

Page | 64
502) we put the pin very close to line angle because this area
a) less material of restoration need
b) intiate dentin crises
c) needs less condinsation of material++++

503) proxy brush used in which of this Furcation sites :


a) Furcation Grade I
b) Grade II
c) Grade III
d) Grade IV

504) pt have inflammation under denture which best tech to take impression
a) take it immediately to prevent future deterioration
b) reline the denture by soft material & wait until tissue is healed ++++
c) advice pt not to remove denture at nigth
d) a & b
e) all of the above

505) successful endo prevents


a) fibrous growth++++
b) bone deposition

506) Pt came with bristle even on mucous membrane u asked for immune test
a) pemphgus
b) bullos pemphogoid++++
c) lichen planus

507) Cavernous Sinus Thrombosis (CST) is not manifested as


a) infra orbital syndrome
b) syncope due to artrial obliteration ++++
c) eye exopthalmous

508) in FPD you use GIC for cementation what is best to do


a) remove smear layer by acid to increase adhesion++++
b) don’t varnish because it affect adhesion
c) mixed slowly on small area until become creamy
d) remove excess when it in dough stage
e) All ++ (if present)

Page | 65
509) Remove excess GIC
a) finishing bur immediately
b) finishing bur later
c) sharp knife immediately
d) sharp knife later
e) a,c
f) b,d++++

510) female pt came with endo treated upper central with m, d caries & have incisal abrasion.
porcelain veneer is planned with modification to cover incisal edge. veneer should end
a) fourth lingualy 0.5 mm before centric occlusal++++
b) fourth 1.5 before centric occlusion
c) fifth 1.5 before centric occlusion

511) pt come with sinus tract. you make gp tracing & take radiograph the gp appear in lateral
surface of the root
a) periodontal abscess
b) periodontitis
c) lateral accessory canal++++

512) Which of this is chemicly bonding to teeth


a) GIC +++
b) composite
c) sealant
d) all

513) What’s the tech of endo fill where we use continuous condensation
a) Ultrafill
b) Obtura I
c) Obtura II
d) System-B++++

514) Post-graduate student use mta the prognosis depend on prevent


a) immediate suture
b) disturbance during closure of wound
c) using a flab

515) the cause of black cast which prevent pickling due to


a) over heat
b) contmiunate with gas++
Page | 66
c) incomplee casting

516) post fracture decrease with


a) prefabricated post
b) ready made post
c) casted post++++
d) metal post

517) in sharpness of instrument the angle between face & blade is


a) 50-60
b) 60-70
c) 70-80++++

518) immature tooth has less sensation of cold , hot due to


a) short root
b) incomplete innervations++++
c) wide pulp chamber

519) Pt take 40 cortizone in day of procedure


a) double the dose just day of procedure++++
b) double the dose day of procedure & day after
c) stop the medication

520) pt has anaphylactic shock after take pncillin u have to give him
a) epinephrin i.v++++
b) adrenalin i.m
c) cortizone i.m

521) Not removing calculus leads to periodontal disease due to


a) calculus is the first antigen
b) it accumulate more plaque to its surface++++

522) the first thing to do after take history of pt is


a) radiograph
b) anaesthesia
c) clnincal examination++++

523) pulpities in decidous teeth in radiograph see related to


a) furcation ++++
b) apex of root
Page | 67
c) lateral to root

524) maxilla is formed


a) before mandible
b) same with mandible
c) sligthly after mandible ++++
d) non of the above

525) pt with clift lip & clift palat with missing of teeth this present with
a) treacher collins syndrome
b) von der wider syndrome++++

526) Treacher collins syndrome have chracterstic feature


a) prognathesia of mandible.
b) no ear loss.
c) upward slenting of eye.
d) malar bone malformed or absent++++ (Malar bone=Zygomatic arch).

527) child have tooth which have no moblity but have luxation best treatment
a) acrylic splint++++
b) non rigid fixation
c) rigid fixation

529. distinguishing between right & left canines can be determined


a) because distal concavities are larger-
b) with a line bisecting the facil surface the tip lies distally
c) …..

530. pt came with class IV he had tooth trauma & he brought the fractured segment & on
examination u found that the pulp is not exposed & only u can see dentine, how u manage:
a) to get rid of the fragment & fill with composite
b) to reattach the fragment with composite and latter cover with veneer++++

531. prophylaxis for pediatric pt.


a) amoxicillin 50 mg/kg 1 hr before procedure.

532. best material for direct pulp capping for a primary tooth
a)calcium hydroxide
b)ZOE

Page | 68
c) formecresol

533. Sealants least effective on


a) primary molars
b) 2nd primary molar
c) …

534. diabetic patient came to clinic with pain & swelling & enlarged mandible, on radiograph it
showed moth eaten appearance, your diagnosis is :
a) acute osteomyelitis
b) focal sclerosing osteomyelitis
c) diffuse sclerosing

535. best instrument to locate vibrating line with it is


a) T burnisher
b)……

536. patient came to dentist after previous stressful procedure complaining of burning &
discomfort of his lip on examination u found lesions on the palate, diagnosis is
a) contact dermatitis
b) allergy
c) aphthous ulser
d) herpes simplex (herpeic gingivostomatitis).

537. In primary tooth for restoration before putting the filling u put
a) base
b) calcium hydroxide
c) varnish

538. for root canal treated tooth u choose to put post & amalgam this depends on
a) remaining coronal structure++++
b) root divergence
c) presence of wide root
d) others

539. patient has a bridge of porcelain teeth, the teeth have different colors although same shade
was selected the cause is

Page | 69
540. patient has a palatal torus between hard & soft palate, the major connector of choice is
a) antero-posterior palatal strap
b) u shaped++++
c) posterior palatal strap

541. While u were preparing a canal u did a ledge, then u used EDTA with the file, this may
lead to
a) perforation of the strip
b)…

542. white lesion bilaterally on cheek,& other member in the family has it
a) leukoplakia
b) white sponge nevus

543. adult 20 years male with soft tissue & dental trauma reveals severe pain in soft tissues
with loss of epithelial layers and anterior upper centrals are intruded the diagnosis is:
a) abrasion with luxation
b) erosion with sub luxation
c) ulceration with luxation
d) ulceration with subluxation

544. trigeminal neuralgia treated by carbomizapine, the maximum dose per day divided in
doses is
a)200 mg
b)500mg
c)1000mg
d)1200mg

545. 10 years old child with congenital heart disease came for extraction of his lower 1st molar,
the antibiotic for choice for prevention of Infective Endocarditis is
a) ampicillin 30 mg /kg orally 1hour before procedure
b) cephalixine 50mg/kg orally 1hour before procedure
c) clindamycin 20mg/kg orally 1hour before procedure
d) amoxicillin 50mg/kg orally 1hour before procedure

546. instrument used for scratching stone cast to make postdam:


a)le cron carver++++

Page | 70
547. after final inlay cementation and before complete setting of cement we should:
a-remove occlusal interferences
b-burnishing of peripheries of restoration for more adaptation
c-lowering occlusal surface

548. adding of surfactant to irrigation solution during RCT to increase wettability of canal
walls by:
a-lowering surface tension
b-increasing surface tension
c-passing through dentinal tubules

549. percentage of teeth which show proximal radiolucency extended to half dentine thickness
but without cavitation:
a-10%
b-20%
c-30%
d-40%

550. pt with renal dialysis the best time of dental tx is:


a-1 day before dialysis
b-1 day after dialysis
c-1week after dialysis

551. Pt with renal transplantation came with white elevated lesion on tongue no history of
smoking or tobacco chewing diagnosis is :
a-candidiasis
b-iatrogenic lesion
c-hyperkeratosis
d-stomatitis

552. in order to decrease the gastric secretion :


a) histamine A antigent
b) histamine B antigent
c) anticholenergic++++
d) adrenal steroids

Page | 71
553. salivary secretion increasing by :
a) cholenergic++++
b) anti diabetic
c) anti cholinergic

554. neonate 2 years old, has a lesion on the ventrum of the tongue... with the eruption of the
1st tooth:
a)Riga-Fede disease <sublingual traumatic ulceration>

555. success of pit & fissure sealants is affected mainly by :


a) increased time of etching
b) contamination of oral saliva++++
c) salivary flow rate
d) proper fissure sealant

556. mechanochemical prep'n during RCT main aim:


a) widening of the apex
b) master cone reaches the radiographic apex
c) proper debridement of the apical part of the canal++++

557. master cone doesn't reach the apex


a) ledge
b) residual remenants (debris)
c) 1&2 ++++

558. the amalgam with the least gamma 2 phase is containing copper in what concent'n

559. vestiloplasty <for thick & wide based frenum> ,


Z-plasty is for thin narrow based frenum.

560. patient came complaining of severe pain on biting, related to a certain tooth. upon
examination no pulpal or periodontal findings, and pulpal vitality is positive, your Dx :

a) cracked tooth syndrome++++

561. cracked enamel best Dx by


a) dye++++

Page | 72
562. pterygomandibular raphe.
a) insertion & origin
b) muscles
c) should be medial to the injection
d) all of the above++++

563. patient with complete denture pronouncing F as a V


a) ant. teeth are upward from lip line ++++

564. child patient with oblitration in the centeral permenant incisor. what will you do :
a)RCT
b)pulpotomy
c)pulpectomy
d)careful monitoring++++

565. patient with 5 years old denture has a severe gag reflex , upon history he says he had the
same symptoms in the first few days of the denture delievery and it went all alone
a) patient has severe gag reflex
b) patient has underlying systemic condition++++
c) denture is overextended

566. all are participating in the determination of the posterior extension of the maxillary
denture (posterior palatal extension) except :
a) hamular notch
b) fovae palatine
c) vibrating line
d) retromolar (pads) areas++++

567. a drawing <STUPID DRAWING>


the divergence mesial & distal for an amalgam restoration:
a) no, it should be convergent
b) yes , if the remaining proximal marginal ridge = 1.6 mm
c) only if > 1.6++++
d) only if < 1.6

Page | 73
568. patient after ortho treatment on x-ray there's resorption <didn't say internal or external> in
the middle of the root , what will you do :
a) fill the resorption with Ca(OH)2++++
b) extract and reimplant
c) extract and do implant
d) do one visit rct

569. patient that has a centeral incisor with severe resorption and who's going through an ortho
treatment that is going to make him extract the premolars, which of the following won't be
present in the treatment plan
a) rpd
b) implant
c) maryland bridge
d) auto implant of the premolars++++

570. patient with radiopacity in the periapical area of a 1st mandibular molar with a wide
carious lesion and a bad periodontal condition is
a) condensing osteitis++++
b) hypercementosis
c) ...

571. patient had anaphylactic shock due to penicillin injection , what's the most important in
the emergency treatment to do :
a) 200 mg hydrocortizone intravenous
b) 0.5 mg epinephrine of 1/10000 intra venous++++ <the only IV in the choices>
c) adrenaline of 1/1000 intra muscular

572. which is contraindicated to the general anaesthia :


a) patient with an advanced medical condition like cardiac ....++++
b) down's syndrome patient
c) child with multiple carious lesion in most of his dentition
d) child who needs dental care, but who's uncooperative, fearful...etc.

573. Squamous cell carcinoma early manifestation appears as :


a. ulcer ++++
b. cauliflower appearance
c. ....
d. ....

Page | 74
574. During 3/4 crown preparation on premolar, bur used to add retentive grooves is
a) Radial fissure++++

575. on a centeral incisor recieving a full ceramic restoration ,during finishing of the shoulder
finish line subgingivally
a) diamond end cutting++++

576. in a class iii composite with a liner underneath, what's the best to use
a)Light cured GI++++
b) ZnO Eug
c) Reinforced ZnOEug

577. patient who has un-modified class II kennedy classification, with good peridontal
condition and no carious lesion the best clasp to use on the other side <teeth side>
a)reciprocal clasp (aker's clasp)++++
b)ring clasp
c)embrasure clasp

578. Which of the following conditions is highly indicated for the short therapy of DOTS and
is directly observed once in the clinic:
a) Tuberculosis++++
b) HIV
c) H1N1
d) Mental Illness

579. At the begining of the Operation day in the clinic, you should start the water/air spray for
three minutes in order to get rid of which type of microorganisms :
a) streptococcus mutans.
b) streptococcus salivaris.
c) ....
d) ....

580. Tooth with a fracture between the apical and the middle third, what's your management
a) RCT for the coronal part only.++++
b) RCT for both.
c) Splint the two parts together.

Page | 75
581. Tooth number 26, had a root canal treatment since two years, upon x-ray you found a
radiolucency with bone resorption along one of the roots.
a. Ca(OH)2.
b. resection of the whole root.
c. redo RCT.
d. periodontal currettage.

582. Stock trays compared to Custom trays for a removable partial denture impression
a. custom trays less effective than stock trays
b. custom trays can record an alginate impression as well as elastomeric impression
c. custom trays provide even thickness of impression material++++
d. all of the above

583. hard palate consists of the following:


a. palatal maxillray process & Ethmoid bone
b. palatal maxillary process & Sphenoid bone
c. palatal maxillary process & Palatine bone++++
d. palatal maxillary process & Temporal bone

584. the main link between the pulp and the the periodontium is:
a. apical foramen
b. dentinal tubules
c. accessory canals
d. PDL

585. the following factors effect the health


1- heriditary
2- environement
3- social and economic factors
4- family welfare
a) 1+2
b) 1+2+4
c) 1+2+3
d) all of the above++++

586. which root of the maxillary teeth is closest to the maxillary sinus:
a. first maxillary premolar
b. maxillary canine
c. 1st maxillary molar++++

Page | 76
587. patient is diagnosed for ceramometal full veneer. you plan to use epoxy
resin >>>>>>missing text>>>>>>>, what's the best impression material to be used :
a. poly ether.
b. poly sulfide.
c. agar agar.
d. irreversible hydrocolloid.

588. patient came with severe pain related to right 1st mandibular molar, there's no swelling
related, pulp test is negative, no evidence in radiograph. Diagnosis:
a. irreversible pulpitis
b. acute peridontal abscess++++
c. suppurative periodontal abscess

589. patient came to your clinic with severe pain, on x-ray the right side of the mandible has a
radiolucency with a radiopaque border that resembles the sunshine rays. your diagnosis is :
a) ossifying fibroma
b) osteosarcoma
c) acute osteomylitis

590. which of the following teeth has a contact area between the incisal (occlusal) third and
middle third
a. 1st maxillary premolar
b. 1st mandibular premolar
c. 1st maxillary molar
d. centeral mandib. Incisor

591. patient returned to you after 1 month from doing amalgam filling with difinite severe pain,
due to:
a. contamination with moisture leading to amalgam expansion
b. unidentified pulpal exposure
c. supra occlusion
d. gingival abscess

592. to plan the line-angles in the proximal cavity in a class II you use:
a. straight chisel
b. biangled chisel++++
c. enamel hatchet

593. after delivery of a complete denture, the patient returned complaining of his phonetics and
pronounciation, upon examination you notice the upper lip is not properly supported.
Page | 77
a. decreased vertical dimension
b. malposition of the anterior teeth++++
c. defficiency in vitamin B

594. NaOCl is used in RCT:


a. oxidative effect
b. ordinary irrigant solution
c. better used diluted
d. better result whend used combined with alcohol

595. 6 years old patient recieved trauma in his maxillary primary incisor, the tooth is intruded.
the permenant incisors are expected to have:
a. yellowish or whitish discoloration
b. displacement
c. malformation
d. cracks in enamel
e. yellowish or whitish discoloration with hypoplasia

596. a 21 years old patient who has iron deficiency anaemia, difficulty in swallowing, with
examination of barium sulphate, you found >>>>>>>missing part>>>>>>>
a. geographical tongue
b. burning mouth syndrome
c. XXXXXXXX syndrome <can't remember the name>
d. diabetec patient

597. the 1st appearance of syphilis


a) Ulcer++++

598. At which temperature that gutta percha reach the alpha temp:
a- 42-48 c++++++++++++++++++++++++
b- 50-60c
c- 70-80c
d- 100c

599. Periodontium consists of:


a- Gingiva – periodontal ligament- aleveolar bone-Cementum++++

Page | 78
600. The x ray show scattered radioopaque line in the mandible jaw the diagnosis will be:
a- Paget disease
b- Garres syndrome
c- Fibrous dysplasia
d- Osteosarcoma

601. All these shows honey combed bone radiographically except:


a- Ameloblastoma
b- Odontogenic myxoma cyst
c- Odontogenic keratocyst
d- Adenomenated tumor++++
Source : http://www.head-face-med.com/content/1/1/3

602. Flouride amount in water should be: (the exact is 0.7 to 1.2 mg/l with max of 1.5)
a- 0.2-0.5 mg/liter
b- 1-5 mg/liter
c- 1-2 mg/liter++++
d- 0.1- 0.2 mg/liter

603. Most common type of cyst:


a- Radicular cyst++++

604. the moist autoclave reaches 100c compared the dry heat :
a- Lesser time++++
b- Longer time

605. 6 artificial mand ant teeth opposes maxillary (dental decks)


a) resorption of the mandible anteriorly++++

606. what is the maximum time for amalgam till to be to its place after tirturation:
a- 1min
b- 3min++++
c- 6min
d- 8min

607. The most superior way to test the vitality of the tooth with:
a- Ice pack++
b- Chloro ethyl
c- Endo special ice++++
d- Cold water spray
Page | 79
608. The way to remove mucocele is
a- Radiation
b- Excision++++
c- Chemotherapy
d- Caterization

609. Functional cusp reduction of onlay:


a- 0.5
b- 0.1
c- 1.5 ++++
d- 2

610. Gingival retraction :


1- To expose the sub gingival area to take good impression to the uncutted portion of the tooth
2- To prepare the tooth to the final impression
3- Necessary even if there’s inflamation
4- Has several methods but the most popular is the retraction cord
a) 1&2 correct
b) 1 only
c) 1, 2 and 4++++
d) 2,3

611. All these are contraindicated to RCT except:


a- Non restorable tooth
b- Vertical root fracture
c- Tooth with insufficient tooth support
d- Pt who has diabetes or hypertension++++

612. Sharpening the curette and sickle, the cutting edge should be at angle:
a- 50-60
b- 70-80++++
c- 80-90
d- 60-70

613. Hypersensitivity is due to:


a- Exposed dentine with opened dentinal tubules++++
b- Obliterated dentinal tubule

Page | 80
614. Avulsed tooth is washed with tap water , it should be replaced again:
a- Immediately++++
b- After 2 hours
c- 24 hr’s
NOTE IT SHOULDN'T BE WASHED WITH TAP WATER ASASAN+++++++++++

615. A 10 yr’s old child, who is unable to differentiate the colors, and cant tell his name, or
address
He is acting like :
a) 3 years old+++++
b) 4 years old
c) 10 years old

616. Patient presented to u complain of click during open and close. Thers is no facial
asymmetry except when opening , What is the diagnosis
a) internal derangement with reduction++++++
b) internal derangement without reduction
c) reumatoid arthritis

617. To check a perforation in the desk of the tmj we need:


a) cranial imagery
b) arthrography++++ (CT after injection of a high contrast fluid)
c) traditional tomography
d) computerized tomography++++

618. Patient have unilateral fracture of left the condyle the mandible will
a)deviate to the left side++++
b)deviate to the right sige
c)no deviate

619. Child has a habit of finger sucking and starts to show orodental changes, the child needs:
a- Early appliance++++
b- Psychological therapy

620. Amount of daily mercury ingestion in the body due to wear of amalgam is:
a) 1-3 µg.++++
b) 3-5 µg.
c) ……

Page | 81
621. Patient comes to your clinic complaining that the denture become tight, during examination you
notice nothing, but when the patient stand you notice that his legs bowing (curved). What you
suspect:
a) Paget’s disease.++++
b) …….
c) …….

622. A 55 year old patient with multi-extraction teeth, after extraction what will you do first:
a) Suturing.
b) Primary closure should be obtained if there is no luntant tissue.
c) Alveoplasty should be done in all cases.++++

623. Child with traumatized lip, no tooth mobility, what will you do first:
a) Radiograph to check if there is foreign body.++++
b) Refer to the physician for sensitivity test.
c) ….

624. How do you know if there are 2 canals in the same root:
a) Radiographically with 2 files inside the root.
b) The orifices are close to each other.
c) ……

625. Weeping canal:


a) Calcium hydroxide.++++
b) GP.
c) ……

626. Patient with lupus erythematous and under cortisone, he needs to surgical extraction of a tooth.
What should the surgeon instruct the patient:
a) Take half of the cortisone dose at the day of operation.
b) Double the cortisone dose at the day of operation.
c) Take half of the cortisone dose day before and at the day of operation and day after.
d) Double the cortisone dose day before and at the day of operation and day after.

627. Hunter Schreger bands are white and dark lines that appear in:
a) Enamel when view in horizontal ground.
b) Enamel when view in longitudinal ground.++++
c) Dentin when view in horizontal ground.
d) Dentin when view in longitudinal ground.

628. Laser core can be used in curing of composite:


a) ND (YAG).
b) HeNe.
c) Argon / Hallogen led.++++

Page | 82
629. During endo patient is complaining of pain with percussion, no fluid extrusion. What do you
suspect?
a) Apical periodontitis.
b) Secondary apical periodontitis.++++
c) Over instrumentation.
d) Over medication.

630. To drain submandibular abscess:


a) Intraorally through the mylohyoid muscles.
b) Extraorally under the chin.
c) Extraorally at the most purulent site.
d) Extraorally at the lower border of the mandible. ++++

631. The aim of biomechanical process during endo is to allow:


a) GP reach the apex. ++++ (removal of all infected pulp and dentine & deprid material)
b) Debridement materials reach the apical area.

632. The aim of treatment maintenance is:


a) Prevent secondary infection.++++
b) Check tissue response.

633. Systemic treatment of oral candida:


a) Nystatin. (topical)
b) Amphotericin.
c) Fluconazole.++++ (SYSTEMIC)

634. Most tooth surface affected by caries:


a) Pit and fissure.++++
b) Root surface.
c) Proximal surface.

635. the impression material of choice when we want to take impression for epoxy resin pin is:
a) Polysulfide.
b) Polyether.
c) …….

636. Streptococcus activity detected by:


a) Fermentation.++++
b) Catalase.

637. Tooth with full crown need RCT, you did the RCT through the crown, what is the best
restoration to maintain the resistance of the crown:
a) Glass ionomer resin with definite restoration.++++

Page | 83
638. A question about Cleidocranial dysostosis characteristic:
• Partly or completely missing collarbones. If the collarbones are completely missing or
reduced to small vestiges, this allows hypermobility of the shoulders including ability to
touch the shoulders together in front of the chest.
• A soft spot or larger soft area in the top of the head where the fontanelle failed to close.
• Bones and joints are underdeveloped. People are shorter and their frames are smaller than
their siblings who do not have the condition.
• The permanent teeth include supernumerary teeth. Unless these supernumeraries are
reabsorbed before adolescence, they will crowd the adult teeth in what already may be an
underdeveloped jaw. In that case, the supernumeraries will probably need to be removed to
provide space for the adult teeth.
• Permanent teeth not erupting
• Bossing (bulging) of the forehead.
• Hypertelorism

639. Cantilever design should has all except:


a) Small in all direction.
b) High yield strength material.
c) Short occlosogingival.++++ (he means the abutment here)
d) Under occlusal.

640. To hasten Zinc Oxide cement, you add:


a) Zinc sulfide.
b) Barium sulfide.
c) Zinc acetate ++++ (Already found in the Catalyst tube in the ZnO Eug paste)
d) Barium chloride.

641. Endomethasone is a root canal sealer that:


a) Dissolve in fluid so it weakens the root canal filling.
b) Very toxic contain formaldehyde.
c) Contain corticosteroids.
d) All the above.++++

642. Child came to the clinic with amalgam restoration fracture at isthmus portion, this fracture
due to:
a) Wide preparation at isthmus.
b) High occlusal.
c) shallow preparation <no beveling for axiopulpal line angle>
d) constricted isthmus <narrow neck of the isthmus>
<the answers written in ITALIC format are added by me, because they are the CAUSES of the
fractured isthmus in class II amalgam preparation.>

Page | 84
643. Child with mental disorder suffer from orofacial trauma, brought to the hospital by his parents,
the child is panic and Irritable, the treatment should done under:
a) Local anesthesia.
b) General anesthesia.++++
c) Gas sedation.
d) Intravenous sedation.

644. Fracture before 1 year of upper central incisor reach the pulp in 8 year old child. how will you
manage this case
a) RCT.
b) Apexification.++++
c) Direct pulp capping
d) Indirect pulp capping.

645. To remove a broken periodontal instrument from the gingival sulcus:


a) Schwartz Periotriever.++++

646. Reliability of the measurements reflects that property of the measurements which
a) Measures what is intended to be measured.
b) Produces repeatedly the same result under a variety of conditions.
c) Detects reasonably small shifts, in either direction, in group condition.
d) All of the above.++++

647. The rows show “truth”, the column show “test result”o
a) Cell A has true positive sample.++++
b) Cell A has true negative sample.
c) Cell A has false positive sample.
d) Cell A has false negative sample

648. indirect retainers mostly needed :


a. class IV (NEEDED ONLY ON VERY LONG SPAN)
b. class I++++
c. class III
d. class III with modification

649. lingual plate :


a. shallow sulcus
b. mobile anterior teeth
c. deep sulcus

Page | 85
650. which palatal form is more retentive and offers better stability to complete denture :
a. v shaped
b. wide palate++++
c. u shaped
d. flat palate

651. biological width


a. 1 mm
b. 2mm++++ (exactly 2.03 mm)
c. 3mm
d. 4mm

652. dentine permeability increases


a. coronal less than root dentine
b. permeablity increase toward DEJ++++
c. permeablity increase toward BCJ

653. upper teeth palatal mucosa supplied by :


a. nasopalatine
b. anterior palatine
c. both
d. post superior alveolar nerve

654. pt with severe pain in lower left mandibular molar, examination positive pulp test , percussion
test, no radiographic abnormality, Rt side has recent fpd upper :
a. chronic apical periodontits
b. actue apical periodontitis
c. apical abcess
d. None of the above

655. epithelial cells


a. rest of malassez decrease with age
b. rest of malassez increase with age
c. hertwig sheath entirely disappear after dentinogenesis
d. epithelial remnants could proliferate to periapical granuloma

656. Pt with complete denture complain from tigthness of denture in morning then become good this
due to
a) relif of denture++++ (because there may be pressure points or areas that the tissues will try to
adjust to it throughout the day)
b)lack of cheeck elastisty (pressure on the flanges <> displacement of denture)
c)poor post dam (no posterior seal <> displacement of denture)

Page | 86
657. Pt construct for him a complete denture after few days he came to u complaining from pain &
white spots on the residual ridgeu do relife in that area & give him ointment & after few days he
came again complaining the same but in another area the main cause is :
a. uneven pressure on the crest of alveolar ridge++++
b. increase vertical dimension

658. To prevent gingival injury place the margin of the ponetic:2.


a. at the level of gingival crest
b.above gigival crest
c. apical to g . crest 1 mm
d. apical to g. Crest ...... mm

659. Aborder line diabetic pt came with denture stomatitis you find abrundant depris in the tissue
surface area of the denture>>the proper managgement is:
a. systemic antibiotic
b. topical antifungal++++ (topical + relining with a tissue conditioner + rest of tissues at night +
good oral hygiene)
c. systemic antifungal
d. topical antibiotic

660. in a curved root u bent a file by4.


a. put guaze on the file & bend it by hand++++ (to avoid finger injury/infection by accidental
piercing of the file)
b. bend the file by plier
c by bare fingure
d. by tweezer

661. pain in central incisors from


a. central &lateral incisors++++
b. lateral & canine
c. canine & premolar
d. premolar & molar

662. to treat non vital tooth with open apex when doing access openning with gates glidden bur take
care to :
a. remove all dentin
b. remove minimal dentine
c. follow conservative method

663. child with previuos history of minor truama with excessive bleeding we do test the result is
prolonged PT & slightly increse clooting time& .........test is positive the diagnosis is
a. hemophelia b
b. thrombocytopenia
c. deficincy in vit. K

Page | 87
664. 20 years old pt have avulsed tooth for 60 min the managment to retun vascularity of the tooth:
a. scrap the surface of the root (NEVER DO THIS)
b. place the tooth in sodium sulfideof X%....
c.place it in sodium chloride then sodium sulfide

665. Pt need complete denture u take impression with irreversible hydrocolloid & poured it after late
more than 15 min the cast appear sort & chalky the reason is:
a. dehydration of the impression
b. expansion of the impression
c. immersion the impression in a chemical solution ++++
(irreversible hydrocolloids = Alginate, a disinfectant as sodium hypochlorite or dettol can affect the
surface details of the alginate if the timing was not correct <more than 10 min’s> )

666. A 60-year-old man has been treated for a T2NOMOsquamous cell carcinoma by radical
radiotherapy. He has a history of chronic alcoholism and was a heavy smoker. Six years after
treatment, he develops apainful ulcer in the alveolar mucosa in the treated area following minor
trauma. His pain worsens andthe bone became progressively exposed. He is treated by a partial
mandibular resection with graft
The diagnosis is
a. acute osteomylitis
b. gerre,s osteomylitis
c. osteoradionecrosis
d. chronic osteomylitis

667. after completion of orthodontic treatment he came complaining of pain in 11 tooth radiograph
show resporption in the middle third of the root of 11 wt is the proper managment:
a. apply caoh at the site of resorption++++
b. do r.c t in a single visit
c. extract the tooth & reimplant it
d. extract the tooth & do implantation

Page | 88

You might also like